Космос и астрономия

Ответить в тред Ответить в тред
Тред тупых вопросов №126 Baade edition Аноним 12/04/20 Вск 02:19:08 5576731
Baade.jpg 293Кб, 957x1418
957x1418
Gaia-stellar-po[...].jpg 407Кб, 1920x1920
1920x1920
HSTSWEEPSGalaxy[...].jpg 292Кб, 740x740
740x740
pulsar-3.gif 4764Кб, 980x551
980x551
Тред вопросов о жизни, Вселенной и всём таком.

Спрашиваем то, за что в других местах выдают путёвку в биореактор. Здесь анонимные учёные мирового уровня критически рассмотрят любые гениальные идеи и нарисованные в Paint схемы.

Предыдущий тут: >>555520 (OP)
https://2ch.hk/spc/res/555520.html

Q: Можно быстрее?
A: Можно упасть в пузырь Альбукерке, NASA уже почти надула его.

Q: Я начитался охуительных историй про уфологию, че делать, нам жопа?
A: Да, тебе жопа, можешь сгонять в зогач или куда оттуда пошлют.

Q: Что будет с человеком в вакууме без скафандра / если он упадет на черную дыру / попробует ступить на поверхность газового гиганта/солнца?
A: Он умрёт.

Q: Почему бы не привязать ракету к воздушному шару или стартовать с горы?
A: Космос - это не как высоко, а как быстро, большая часть энергии ракеты уходит на разгон вбок.
Подробнее тут https://what-if.xkcd.com/58/ (английский) https://chtoes.li/orbital-speed/ (перевод)
Аноним 12/04/20 Вск 06:45:35 5576822
>>557673 (OP)
Какая ебанная жалость!
Во первых пузырь Мойшенника.
Во вторых, надувать нужно очень осторожно, иначе будет как с очередным SN.
В третьих пока получается не очень, облета Луны пока не произошло.
Аноним 12/04/20 Вск 09:00:33 5576863
>>557673 (OP)
Пользуясь случаем задам вопрос, классический и для будущего космонавтики и моей гравицапы.

Сколько энергии потребуется для вертикального взлёта некого объекта на высоту 1 000 м весом 100 кг?

То как я это вижу - 1 000 000 Дж.

И второй вопрос.

Вес в Ньютонах объекта 100 кг равен 1 000 Н. Возможно ли вылететь за пределы тяготения Земли, если его будет толкать некий крутой и долгоработающий движок с силой в 1 500 Н?
Аноним 12/04/20 Вск 09:38:58 5576904
>>557673 (OP)
Что с солнцем? Оно и в прям впадет в гибернацию?
Аноним 12/04/20 Вск 09:54:29 5576915
>>557686
>Сколько энергии потребуется для вертикального взлёта некого объекта на высоту 1 000 м весом 100 кг?
При тяговооруженности 1 - бесконечность. Задумайся об этом. и рано или поздно дойдешь до лагранжевой механики самостоятельно
>Вес в Ньютонах объекта 100 кг равен 1 000 Н. Возможно ли вылететь за пределы тяготения Земли, если его будет толкать некий крутой и долгоработающий движок с силой в 1 500 Н?
Да, конечно, в чем проблема? Можешь даже строго вверх тянуть, тебе хватит. Если у тебя изначально объект по орбите движется - достаточно даже 0.0001Н, просто тяни вдоль неё и разгоняйся себе, рано или поздно вылетишь.
Аноним 12/04/20 Вск 10:53:34 5577056
>>557673 (OP)
Ебанный стыд...
Во-первых, Алькубьерре.
Во-вторых, не упасть, а создавать вокруг корабля изнутри (иначе кина не будет).
В-третьих, НАСА искривляет пространство на десятимиллионную часть, контролируя
это сверхточными интерферометрами, до самого варп-привода здесь - как до
Антарктиды раком.
Аноним 12/04/20 Вск 10:54:45 5577067
>>557673 (OP)
>Baade edition
Респект ОПу, кстати. Бааде нечасто вспоминают.
Аноним 12/04/20 Вск 10:57:45 5577098
>>557686
>Вес в Ньютонах объекта 100 кг равен 1 000 Н. Возможно ли вылететь за пределы тяготения Земли, если его будет толкать некий крутой и долгоработающий движок с силой в 1 500 Н?
С одной стороны да, с другой нет. "Волшебный" движок сможет. Реальный же упрется в то, что чем медленнее движок отбрасывает назад топливо, тем больше топлива понадобится взять с соой. Движок на 150 кг тяги сможет поднять 100 кг груза, но не сможет поднять 100 кг груза вместе с 900 кг топлива. В целом, если кидать назад топливо очень быстро, то тогда топлива может понадобиться мало. Скажем, если кидать топливо назад со скоростью ~120 км/c то для вылета за пределы тяготения Земли понадобится порядка 10 кг топлива. Но поскольку тяга с ростом скорости истечения топлива растет линейно, а кинетическая энергия топлива - квадратично, то при сохранении тяги затраты энергии растут линейно. И с такими движками быстро упираемся в то, что жрут они киловатты, а тяга при этом измеряется долями ньютона. Скажем для двигателей уровня СПД, 1500 Н тяги потребуется около 22 мегаватт энергии. При том, что им понядобится более 100 кг топлива, что бы разогнать 100 кг груза до второй космической, с учетом всех потерь. То есть понадобится уже не 1500 Н, а 3000 и уже 44 мегаватта. И после этого вспоминаем, что и движки и генератор весят, и никаких 100 кг уже не остается, счет пойдет на тонны и десятки тонн. И далее по кругу.
Гипотетически проблему бы решил двигатель с внешним питанием - скажем светим лазером в зеркальный парус, который дает 1500 Н тяги. Но пока сделать такие движки не удалось, там при любых реализациях начинают вылезать подводные камни.
Аноним 12/04/20 Вск 11:07:37 5577169
>>557714
>заговорчики в /zog
Я всего лишь хочу пруфов на величайшее событие в истории космонавтики, в честь которого назван сегодняшний день.
Аноним 12/04/20 Вск 11:14:03 55771710
>>557716
Полно пруфов. Когда ты их отрицаешь - тебе место в зогаче.
Аноним 12/04/20 Вск 11:19:23 55771811
>>557717
Принеси хоть один.
Аноним 12/04/20 Вск 11:22:35 55771912
Аноним 12/04/20 Вск 11:30:38 55772113
>>557719
Его дочь говорит что эти кадры сняты потом.
Аноним 12/04/20 Вск 11:32:28 55772214
>>557721
>>557717
>Когда ты их отрицаешь - тебе место в зогаче
Для тебя целая доска, займись там кругодрочем среди таких же имбецилов что и ты.
Аноним 12/04/20 Вск 11:47:51 55772315
Аноним 12/04/20 Вск 11:50:54 55772516
>>557723
https://youtu.be/Wyrj0ibxt8c
>Я все ещё в ожидании пруфов
Пиздуй нахуй отсюда, ты зогачерская мразь, тебе ничего не будет пруфом.
Аноним 12/04/20 Вск 11:51:45 55772617
Что, порашные зогачеры опять пришли перефоршивать луносрач?
Аноним 12/04/20 Вск 12:01:31 55773118
>>557725
Очевидный магнитофон очевиден.
>>557726
Я не сомневаюсь в том что граждане США были на Луне.
Аноним 12/04/20 Вск 12:03:07 55773219
>>557731
Поэтому ты пришёл перефорсивать американский и сам по себе беспруфный лунозаговор на Гагарина, обладая единственным пруфом того, что он там не был, крик "PROOOOOFS!".
Аноним 12/04/20 Вск 12:06:55 55773420
>>557732
Я не собираюсь нечего перефорсивать. Я хочу разобраться, в это событии. В том то и дело, что в полете американцев есть железные пруфы, даже помимо видео, а у полета Гагарина нет ничего, даже видео с ракетой и оказывается фейком.
Аноним 12/04/20 Вск 12:09:01 55773521
>>557734
Пиздуй в зогач, чмо. Ты уже показал, что тебе не нужны пруфы и ты хочешь хуесосить реальность и будешь отрицать все пруфы.
Таких уебков как ты надо четвертовать по-хорошему.
Аноним 12/04/20 Вск 12:11:25 55773622
>>557734
Упорыш, ответь на один простой вопрс. Почему то, что Гагарин не был в космосе, очевидно для тебя, зогачера-порашника, но не было очевидно и указано ни ЦРУ, ни МИ-6, а Гагарин аж с королевой Англии обедал?
Вообще, люблю конспирологов, с их тайными заговорами, о которых знают только масоны, рептилоиды и рандомный хуй с зогача
Аноним 12/04/20 Вск 12:19:54 55773923
>>557737
Не собираюсь метать бисер, свинья, ты уже показал свою сучность.
Аноним 12/04/20 Вск 12:46:57 55774524
Найс, мои посты удалили, где не было ничего кроме вопросов по теме треда, но оставили оскорбления в мой адрес. Я разочарован в модерации спейсача.
Аноним 12/04/20 Вск 12:50:11 55774625
Почему в космос не забрасывают ядерные реакторы? Не ритеги, а нормальные. Можно и защиту тяжеленную не брать, там и так излучения полно. Разве не выгодно заменить поля солнечных панелей на один блок с реактором?
Аноним 12/04/20 Вск 12:55:16 55774726
>>557746
Ну забросишь ты его, а дальше что? Сейчас реактор в космосе не нужен, там нет такой хуйни, которая бы жрала так много энергии
Аноним 12/04/20 Вск 12:56:10 55774827
>>557745
Нехуй быть жирной скотиной.
Пруфы тебе не пруфы и ты это считаешь нормальным поведением?
Аноним 12/04/20 Вск 12:57:10 55775028
>>557746
1) Рядом с Землёй от Солнца хуячит поток более чем в киловатт. Это вполне достаточно для всех потребностей.
2) СБ просты и надёжны. Отказывать там вобщем-то нечему, ресурс гигантский.
3) Реактор сложен и ненадёжен. Сделать турбину, которая бы работала без капремонта и остановки хотя бы 15 лет... Нереально. Сделать огромный радиатор и надеятся, что ничего не утечет за столько времени... Очень оптимистично.
4) Реактор хорош только там, где Солнца мало. Но туда редко летают.
Аноним 12/04/20 Вск 12:57:53 55775129
>>557746
Забрасывали.
Так мощно никто не потребляет энергию чтоб надо было йоба-реактор.
Вблизи хватает солярок. Вдалеке хватает ритэгов.
Как полетим Марс осваивать (ох, когда же) - тогда и реактор зашлют.
Аноним 12/04/20 Вск 13:01:31 55775330
>>557723
Сначала покажи, где она говорит, что он не летал.
Аноним 12/04/20 Вск 13:05:23 55775431
блять, вы заебали шизика кормить. какие же вы тупые. пусть на говно исходит сам с собой

>>557746
Забрасывали. На низкой орбите получилось вот что:
https://ru.wikipedia.org/wiki/Космос-954
После этого этим спутникам срочно запилили катапульту для активной зоны (чтобы в случае схода выбросить какаху на высокую орбиту, откуда очень долго не сойдёт). Потом вообще отказались - опасно слишком.

В дальний космос собираются забрасывать, но вяло и дальше испытаний (в лучшем случае) дело ни у кого не идет. См. например тепловые ядерные двигатели, испытанные на земле. А также ЯЭДУ вроде отмененного Jupiter Icy Moons Explorer или пока вроде не отмененного, но очень вялотекущего ТЭМ. Еще есть планетарные реакторы, см. Kilopower для Марса, опять же естественно никуда не летавший (но хотя бы сделанный в железе).

>>557747
>>557750
Наоборот, энергии люто не хватает. Было б чем охлаждать, а применения всегда найдутся. СБ не так просты как кажутся, тонкие структуры в космосе колышутся, и механически нетривиальны. У достаточно мощного реактора, в свою очередь, нетривиальны радиаторы, потому их и не юзают.
Аноним 12/04/20 Вск 13:07:15 55775532
>>557754
>СБ не так просты как кажутся, тонкие структуры в космосе колышутся, и механически нетривиальны.
Ну и да, СБ тоже радиаторы нужны, хотя и намного более простые, т.к. СБ слишком чувствительны к диапазону температур.
Аноним 12/04/20 Вск 13:09:24 55775733
>>557754
>Jupiter Icy Moons Explorer
Orbiter, конечно же. В этом и была его суть, покрутиться вокруг каждой луны.
Аноним 12/04/20 Вск 13:11:16 55775834
>>557673 (OP)
sarcasm.jpg
Как аы объясните, что Гагарин полетел именно в день космонавтики, а не в любой другой день? Явно же постановка, чтоб он к празднику полетел!
Аноним 12/04/20 Вск 13:12:28 55775935
>>557748
Ещё раз, пруфов принесли два- видео с ракетой, и запись переговоров. По факту первое видео оказалось снятым постфактум, ну а второе легко фальсифицируемо, бытовым магнитофоном. Почему я предполагаю факт фальсификации? Например потому что незадолго до этого маршал Неделин разбился в авиакатастрофе
Аноним 12/04/20 Вск 13:13:54 55776136
Смех Гоблина на[...].mp4 1455Кб, 1280x720, 00:00:10
1280x720
>>557758
Спасибо, нечасто искренне смеюсь от души так.
Аноним 12/04/20 Вск 13:14:38 55776237
>>557748
>Нехуй быть жирной скотиной
И моча, обрати внимание, опять в мою сторону оскорбления, которые не провоцировались мной.
Аноним 12/04/20 Вск 13:15:47 55776338
>>557759
Пруфы принесли, мразь.
Ты не заинтересован в фактах, ты хочешь спорить с реальностью.
Точно так же можно себя вести по отношению к любому вопросу, гной.
ВОДА НЕ ЖИДКАЯ ВЫ ВСЕ ВРЁТИ.
ДОКАЖИТЕ ЧТО ВОДА ЖИДКАЯ
>пруф
НЕТ ВРЁТИ, ЭТО НЕ ТАК ВСЕ

Какой же ты уёбок.
Аноним 12/04/20 Вск 13:16:18 55776439
>>557762
Какого хуя ты вообще постишь, тебе пермач надо выдать, гнида ебаная.
Аноним 12/04/20 Вск 13:24:40 55776640
>>557763
>>557764
>Ты не заинтересован в фактах, ты хочешь спорить с реальностью.
Из тебя плохой телепат
>мразь
>гной
>уёбок
>гнида ебаная
Если это единственное чем ты можешь подтвердить его полет, то ты сказал достаточно, можешь не продолжать.
Аноним 12/04/20 Вск 13:32:05 55776741
>>557747
>>557750
>>557751
>не нужно столько электричества
А испытывать всякую электродвигательную йобу типа васимра?
>>557754
Мда. Про спуск с орбиты я как-то не подумал.
Аноним 12/04/20 Вск 13:40:12 55777042
>>557767
Ну ЯЭДУ и питают "ЭДУ", электродвигательную установку, будь то вазимр или еще что. А вообще энергия дохуя на что нужна кроме движков, связь прорву жрёт и работает тем лучше чем больше энергии, SAR тоже, любая сложная электроника, разнообразные телескопы, в общем был бы источник, а применений полно.
Аноним 12/04/20 Вск 13:45:45 55777243
>>557766
Пошёл на хуй, чмо.
Аноним 12/04/20 Вск 14:05:38 55777844
Самое забавное, что Гагарин действительно не был первым космонавтом - так как не совершил полный оборот. Он был первым астронавтом - первым поднялся выше 100 км. Первым космонавтом де-юре был Титов
Аноним 12/04/20 Вск 14:06:31 55777945
>>557778
Сука, как ты до сих пор не в бане, пиздливое хуйло?
Аноним 12/04/20 Вск 14:08:26 55778046
>>557779
Но я не >>557766
Я не говорю, что Гагарин не был в космосе. Был, разумеется.
Я говорю, что он не был космонавтом по классификации НАСА, но был астронавтом.
Аноним 12/04/20 Вск 14:12:41 55778147
>>557780
>>557778
>не совершил полный оборот
Перетолстил.
Аноним 12/04/20 Вск 14:15:32 55778548
>>557781
Он приземлился где-то градусов на 10 западнее Байконура.
Аноним 12/04/20 Вск 14:19:12 55778649
94TwHIBnCs.jpg 102Кб, 540x300
540x300
Он летел как-то вот так. Чисто технически всё-таки оборот не полный
Аноним 12/04/20 Вск 14:28:51 55778850
>>557786
Вокруг земли или вокруг поверхности земли.
Аноним 12/04/20 Вск 14:31:58 55778951
>>557786
Относительно центра оборот был полный. Он же там дольше суток был, Земля повернуться успела.
Аноним 12/04/20 Вск 14:54:45 55779252
>>557789
Очень сильно ошибся.
>>557786
Но да, планета-то успела повернуться, так что в замороженной системе координат был полный виток.
Аноним 12/04/20 Вск 15:01:52 55779453
>>557786
Ага. А спутники на ГСО вообще тогда витков не совершают?
Аноним 12/04/20 Вск 15:04:23 55779554
>>557759
>маршал Неделин разбился в авиакатастрофе
А Гагарин погиб при взрыве ракеты.
Аноним 12/04/20 Вск 15:04:37 55779655
Насколько я понял вся эта белеберда с классификацией космонавтов связана с космической гонкой и непризнанием\признанием Шепарда. Так ли это?
Вроде бы были пуски людей на р7 не достигающие 100 км. Были ли? Какой высоты достигли?
Аноним 12/04/20 Вск 15:06:48 55779756
>>557796
Шепард даже на орбиту не вышел, его полёт продлился меньше 15 минут. Первый оборот вокруг поверхности был у Титова.
Аноним 12/04/20 Вск 15:15:02 55780057
>>557673 (OP)
>Gaia-stellar-populationsjpg
И что это значит?
Аноним 12/04/20 Вск 15:18:51 55780258
>>557780
Это писал не я.
>>557795
Я не просто так выделил этот фрагмент курсивом.
Аноним 12/04/20 Вск 15:56:11 55781659
>>557780
>Я говорю, что он не был космонавтом по классификации НАСА, но был астронавтом.
У НАСА нет классификации космонавта блядь.

>>557778
>не совершил полный оборот
Совершил.

>де-юре
В какой юрисдикции? Так-то, если уж действительно хочется разбирать полёты де юре, то по определению ФАИ полёт Гагарина не должен быть засчитан, потому что он не приземлился внутри капсулы, а катапультировался (именно поэтому необходимость катапультирования из Востоков долго скрывали). А первым в таком случае был Джон Гленн.

>>557789
>Он же там дольше суток был
Охуительные истории.
Аноним 12/04/20 Вск 15:59:24 55781860
>>557816
>>Он же там дольше суток был
>Охуительные истории.
108 часов это меньше суток?
Аноним 12/04/20 Вск 16:01:43 55782061
>>557818
Нет, 108 часов - это 4,5 суток. Спасибо, что проконсультировались с ТТВ.
Аноним 12/04/20 Вск 16:45:21 55785562
>>557786
Чисто технически количество совершенных обротов неважно. Важен факт выхода на замкнутую орбиту. Сход осуществлялся запуском тормозной установки. Если бы не сработала, то подразумевалось что ресурсов системы жизнеобеспечения хватит на сход с орбиты торможением об атмосферу, дней эдак за 5. Правда при запуске Гагарина орбита получилась чуть выше запланированной, и при отказе тормозного двигателя он бы умер на орбите.
У Шаттла тоже, кстати, тоже был предусмотрен вариант миссии, при котором он сходил с орбиты на первом же витке, при запуске военных сппутников на полярную орбиту. При таком полете в космосе он тоже проводил меньше полного витка. Правда после катастрофы Челленджера вояки сказали "ну нахуй" и на полярные орбиты Шаттл так и не летал.
Аноним 12/04/20 Вск 16:46:50 55785663
>>557818
Гагарин был в космосе 108 минут, а не часов.
Аноним 12/04/20 Вск 16:55:37 55785864
>>557800
Что в Млечном пути существуют две разных звездных популяции:
— старые звезды, образовавшиеся вскоре после Большого взрыва и почти не содержащие тяжелых элементов, распространены в основном в центральном балдже и в галактическом гало.
— и относительно молодые звезды, образовавшиеся уже после того, как в межзвездный газ сверхновые миллиарды лет срали тяжелыми элементами. В Млечном пути они в основном распространены в галактических рукавах, типичный пример — Солнце.

Чего ты доебался вообще, знаешь, как сложно найти не уродливые картинки, подходящие под тему треда?
Аноним 12/04/20 Вск 16:59:57 55786065
>>557816
>катапультировался
Кто-нибудь, объясните вкратце, почему нужно было катапультироваться вместо нормального приземления.
Аноним 12/04/20 Вск 17:06:21 55786366
>>557860
Так в сценарии было.
Аноним 12/04/20 Вск 17:07:06 55786467
>>557860
Отрабатывали атомную бомбардировку Воронежа Вашингтона.
Аноним 12/04/20 Вск 17:13:24 55786968
>>557860
У Востока не было двигателей мягкой посадки и он весьма сильно бухался об землю при приземлении. Посадку внутри корабля космонавт скорее всего пережил бы, но вполне мог получить травмы.
Аноним 12/04/20 Вск 17:14:13 55787169
>>557869
> но вполне мог получить травмы
У нас что, космонавтов мало было?
Аноним 12/04/20 Вск 17:22:33 55787670
>>557871
На тот момент один.
Аноним 12/04/20 Вск 17:39:24 55788571
>>557869
>У Востока не было двигателей мягкой посадки
В сраном совке даже двигатели мягкой посадки были в дефиците.
Аноним 12/04/20 Вск 19:18:17 55790372
15861820407860.png 1285Кб, 1920x1080
1920x1080
Насколько близко планета может пролететь к звезде? Вот например берём железо-каменную планету уровня Меркурия и отправляем её с ебеней за Плутоном направляем к солнышку. Насколько близко такая планета сможет подлететь к солнцу, чтобы суметь вернуться обратно?
Сможет ли такая планет нырнуть в самые вершины атмосферы? А нырнуть под поверхность звезды и вынырнуть, оставшись на траектории?
А сможет ли такая планета стать полностью расплавленной? И сможет ли факт превращения твёрдого тела в жидкость (пускай густую и вязкую), кардинально изменить траекторию движения?
Аноним 12/04/20 Вск 19:28:25 55790873
>>557673 (OP)
НАСА хочет во влажных фантазиях отправить на луну рой роверов размером с мыло 10х10х5 см. В чем профит и какие у них плюсы?
Аноним 12/04/20 Вск 19:32:02 55790974
>>557903
Есть предел Роша для жидких тел, ближе него не подлететь. Планета удерживает свою форму за счет самогравитации, и подлетев достаточно близко к другому телу, его просто распидорасят приливные силы, которые пересилят эту самогравитацию.

>сможет ли факт превращения твёрдого тела в жидкость (пускай густую и вязкую), кардинально изменить траекторию движения?
Даже если магически нагреть до плавления - конечно нет, система-то все равно замкнутая. (не считая давления излучения, исходящего из нагретого тела, но оно во все стороны идёт). Больше того, в макромасштабах любое круглое астрономическое тело и без расплавления представляет собой каплю жидкости - самогравитация пересиливает прочность любых анобтаниумов, потому оно и круглое.

Алсо, есть например экзопланета NGTS-10b, она вращается практически около звезды, на расстоянии в 1.5 предела Роша, один оборот за 18 часов. Ещё ближе и её просто разорвет на куски, и она станет кольцом мусора вокруг звезды.
https://arxiv.org/abs/1909.12424
Аноним 12/04/20 Вск 19:33:45 55791075
>>557860
Слишком дохуя весили парашюты для всей капсулы целиком, поэтому решили делать только для кресла. Позже для союза придумали систему реактивной посадки.
Аноним 12/04/20 Вск 19:37:44 55791176
>>557903
Зависит от того, насколько планета может удерживать свою форму, чтобы её не разорвало приливными силами, гугли предел Роша. Для Солнца предел Роша для абсолютно жёсткого тела находится под поверхностью, а для жидкого тела - намного дальше. Но поскольку при подлёте планета начнёт плавиться, то скорее всего занырнуть не сможет. Но для более раздутой звезды, например красного гиганта, может и возможно. Алсо, стоит помнить, что звёзды состоят из газа, у них, как и у Юпитера, нет чёткой поверхности.
Аноним 12/04/20 Вск 19:44:21 55791577
>>557911
> Для Солнца предел Роша для абсолютно жёсткого тела находится под поверхностью
*это для планеты примерно земной плотности, у Меркурия такая же.
Аноним 12/04/20 Вск 19:46:44 55791678
Аноним 12/04/20 Вск 19:47:07 55791779
>>557911
>>557915
Предел для жёсткого тела не применим к планетам и вообще к любым телам, достаточно большим для самостоятельного сохранения круглой формы. Т.к. они не жесткие тела изначально.
Аноним 12/04/20 Вск 19:48:06 55791880
>>557911
Как это у юпитера нет поверхности? Он из газа состоит?
Аноним 12/04/20 Вск 19:48:57 55791981
>>557916
Даже так. Профит от мелко роверов какой?
Аноним 12/04/20 Вск 19:51:49 55792182
>>557918
Нет четкого раздела между средами. Будешь тонуть как в болоте, пока тебя не утрамбует в металлический водород.

На Венере есть что-то слегка похожее. Там есть твердая поверхность, но атмосфера внизу такая плотная и горячая, что углекислый газ находится в сверхкритическом состоянии, т.е. нет разницы между газом и жидкостью.
Аноним 12/04/20 Вск 19:54:09 55792383
>>557921
>Будешь тонуть как в болоте, пока тебя не утрамбует в металлический водород.
Чот подозреваю, что Архимед с тобой не согласится.
Аноним 12/04/20 Вск 19:57:52 55792484
>>557917
Да, действительно. Ну тогда точно не сможет. Но например для Бетельгейзе даже для жидкого тела предел Роша под поверхностью.
Аноним 12/04/20 Вск 20:04:55 55792585
>>557923
Хотя нет, ты пожалуй прав.
>Beneath that, where the planet’s hydrogen are in a liquid state, the density rises to roughly 0.5 g/cm³ and increases to 1 g/cm³ at the boundary with the layer composed of metallic hydrogen.
Аноним 12/04/20 Вск 20:15:06 55792686
>>557924
>Но например для Бетельгейзе даже для жидкого тела предел Роша под поверхностью.
Как это так? Невозможно
Аноним 12/04/20 Вск 20:16:08 55792787
>>557709
>>557691

Понял.

Ещё вопрос.

Как известно, или мало известно, звуковые волны способны создавать давление, так же я где-то слышал, что подобно колесу, в зависимости от частоты вращения которого относительно камеры, что снимает 25 кадров в секунду, для нас оно крутится с разной скоростью, так и для этих волн в некий момент времени итоговая волна будет смещаться взад. вперёд или стоять на месте.

Возможно ли создание суперпозиций звуковых волн таких чтобы -они толкали, притягивали, причём без необходимости переотражения от поверхности Земли, а благодаря их взаимному столкновению?
Аноним 12/04/20 Вск 20:19:24 55792888
>>557926
Возможно, если кроме гравитации звездную плазму удерживает ещё что-то например ебучие магнитные поля
Аноним 12/04/20 Вск 20:23:42 55793089
>>557926
Почему невозможно? Посчитай сам. Вообще, если посмотреть на формулу, то там отношение плотностей в степени 1/3, а радиус (звезды в данном случае) в первой степени. То есть, если например раздуть Солнце в два раза, то плотность его уменьшится в 8 раз, но в формуле степень 1/3, то есть это отношение уменьшится в 2 раза, то есть предел Роша не зависит от растягивания звезды при неизменной массе. Поэтому достаточно растянуть Солнце в 1.55 раз, чтобы предел для жидкого тела был на поверхности. Если растянуть больше, предел окажется под поверхностью. Поэтому в будущем, когда Солнце станет красным гигантом, какая-нибудь из планет возможно сможет летать под поверхностью (ну то есть могла бы, если бы не тормозилась об газ).
Аноним 12/04/20 Вск 20:28:50 55793290
>>557927
Ты описываешь акустическую левитацию. Это возможно и применяется дохуя где.
https://www.youtube.com/watch?v=669AcEBpdsY
Она хорошо работает для небольших объектов. И очень плохо - для больших.
Аноним 12/04/20 Вск 20:36:37 55793391
>>557932
Понял, но тут как я вижу нужно чтобы волна отражалась. А без отражения волн с суперпозицией в пространстве есть?
Аноним 12/04/20 Вск 21:18:34 55795292
unnamed.jpg 30Кб, 900x900
900x900
>>557928
>>557930
То есть, гипотетический Меркурий спокойно бы нырнул и вынырнул не только в атмосферу Бетельгейзе, но под его поверхность, и при этом такому Меркурию было бы в целом норм, как с гуся вода?
Поиграв в КСП, я понял, что чем более из далека выстроена орбита к телу тем более вытянутой она становится, и тем более ускоряется тело при приближении к планете/звезде при своём приближении, значит и такой Меркурий проведёт у звезды (а в случае Бетельгейзе, видимо даже и внутри), очень не долго, пролетая максимально быстро. А это-то влияет на подповерхностное? Получится ли, чт при таком нырке планета нагреется незначительно и лишь парой десятков сотен км от поверхности?

И пока я писал вопрос, родился второй: можно ли ракете или планете выйти на гравитационный манёвр око чёрной дыры так, чтобы при приближении (а следовательно и наборе скорости), объект бы набрал скорость свете или сверхсвет? В случае ракеты можно тягу включить, а в случае планеты кинетическая энергия же как-то должна помочь на таком манёвре
Аноним 12/04/20 Вск 21:42:55 55796593
1586716974410.jpg 220Кб, 872x994
872x994
Назовите хоть одну вменяемую причину, почему не производятся НИОКР клиновоздушных двигателей?

Только не надо тут ко-ко про дорого и т.п.
Не дороже бесполезного слс или спутника-аналогоанета будет.

Зато профиты колоссальны — можно будет увеличить грузоподъёмность ракет в разы блять.
А значит и снизить стоимость во столько же раз.

Это что, заговор рептилоидов для торможения космической экспансии человечества?
Аноним 12/04/20 Вск 21:49:37 55797094
Аноним 12/04/20 Вск 21:55:39 55797495
>>557965
Ты "техникой-молодёжи" под стекломой объебенилась что ли, пидораха? Поднять в разы можно разве что цену, а удельный импульс — на единицы процентов. Никому нахуй не надо с таким говном связываться.
Аноним 12/04/20 Вск 21:57:51 55797596
насколько сильн0 влияние взрыва звезды на её планеты? возмём какую-нибудь землеподобную обитаемую планету у звезды,к оторая взрывается. естественно, в момент взрыва во все стороны летит раскалённое вещество, неся энергию, много энергии в джоулях (или не в них?), но всеь этот поток сброшенной массы же должен пролететь быстро, а значит не так уж и губителен должен быть. просто как стен огня из космоса, длящаяся секунды. вангую, в таком случае сильно подпалило бы растительность на дневной стороне планеты, но так как растения сырые, то есть живые, то кроме ожога листвы в тропических регионах, да поджога торфов в умеренных, ничего бы толком не произошло. ледники и тайги не успели бы за мгновение растаять, где слякоти и болота внутренняя влага быстро бы создала прослойку пара. спася мох и припочвенную растительность, а там, где степи, луга и поля, и пожар бы был неизбежен (особенно если летний горячий сезон), то там бы было как в этом году в австралии - огонь выжигает растительность гектарами, а там через пепел уже прорастает другая
Аноним 12/04/20 Вск 22:19:53 55798297
>>557952
>не только в атмосферу Бетельгейзе, но под его поверхность
Тебе же сказали уже, у звёзд, как и у газовых гигантов, атмосфера плавно перетекает в поверхность. Поверхностью Солнца считается фотосфера - тонкий слой атмосферы, который излучает свет. Она сама постоянно бурлит ( https://en.wikipedia.org/wiki/Granule_(solar_physics) ), а у Бетельгейзе её вообще колбасит очень сильно, и эти гранулы гигантские даже по сравнению с самой звездой.

>Поиграв в КСП, я понял, что чем более из далека выстроена орбита к телу тем более вытянутой она становится, и тем более ускоряется тело при приближении к планете/звезде при своём приближении
Молодец, да, это законы Кеплера.

>значит и такой Меркурий проведёт у звезды (а в случае Бетельгейзе, видимо даже и внутри), очень не долго, пролетая максимально быстро.
Не совсем. Да, если подлетать близко к Солнцу, будешь лететь очень быстро, но чтобы летать внутри, нужно растянуть Солнце, не меняя массу. Тогда и гравитация его не будет меняться, то есть скажем если растянуть Солнце до размеров орбиты Меркурия (а это с ним произойдёт когда-нибудь), то на орбиту Меркурия это никак не повлияет, он всё так же будет вращаться с той же скоростью. Вернее, так было бы, если бы можно было пренебречь излучением Солнца, его атмосферой и т.д. Бетельгейзе вообще гигантская, её радиус где-то в пределах от радиуса орбиты Марса до орбиты Юпитера.

>А это-то влияет на подповерхностное? Получится ли, чт при таком нырке планета нагреется незначительно и лишь парой десятков сотен км от поверхности?
Вообще говоря да, если планета очень быстро пролетает на близком расстоянии от звезды (не пересекая предел Роша и не разрушаясь при этом), то она не успеет прогреться полностью. Так кометы летают например.

>можно ли ракете или планете выйти на гравитационный манёвр око чёрной дыры так, чтобы при приближении (а следовательно и наборе скорости), объект бы набрал скорость свете или сверхсвет
Чтобы набрать скорость света, надо затратить бесконечно много энергии, либо иметь нулевую массу. Чтобы набрать сверхсветовую скорость, нужно надуть пузырь Альбукерке обчитаться сайфая. Вообще, чёрная дыра, по определению, имеет вторую космическую выше скорости света. Набрать околосветовую скорость таким образом можно, действительно за счёт кинетической энергии, то есть нужна или очень быстро вращающаяся ЧД, или тесная двойная (и тех и других в принципе немало). Но опять же много проблем с приливными силами, проще вот так (это эффективно то же самое): https://www.youtube.com/watch?v=rFqL9CkNxXw
Аноним 12/04/20 Вск 22:23:26 55798498
При пролете через атмосферу планета замедлится и упадет на звезду.
Аноним 12/04/20 Вск 22:28:00 55798599
Почему американцы из своих КА выскакивают налегке и весело, а наши еле-еле вылазят из своих аппаратов?
Аноним 12/04/20 Вск 22:35:21 557987100
>>557975
>насколько сильн0 влияние взрыва звезды на её планеты
Там где-то на what-if пробегало сравнение, что взрыв сверхновой на месте Солнца выглядел бы для земного наблюдателя в охулиард раз ярче, чем взрыв кузькиной матери, прижатой к его зрачку. А еще, даже потока нейтрино от взрыва (которые считай что ни с чем не взаимодействуют) достаточно, чтобы убить человека. Так что, думаю, влияние заметное.
Аноним 12/04/20 Вск 22:36:04 557988101
>>557985
Потому что организм ослаблен водкой, да и кто захочет даже временно съебав возвращаться сюда?
Аноним 12/04/20 Вск 22:41:57 557989102
загруженное.png 49Кб, 800x400
800x400
>>557984
А если местная звезда обладает таким сильным внутренним давлением света и таким сильным вращением, что суммарно, в купе с огромной скоростью самой планеты, получится эффект выталкивания звездой планеты? Типа как говно, которе каким бы большим не было на поверхности озера, всё равно оказывается легче воды. Ну или пробка. Или гусь. Или поплавок.
Аноним 12/04/20 Вск 22:43:55 557991103
360px-Купевфирм[...].jpeg 30Кб, 360x240
360x240
Аноним 12/04/20 Вск 22:48:28 557993104
Сможет ли в атмосфере венеры плавать подводная лодка?
Будут ли работать вертолет, самолет?
Аноним 12/04/20 Вск 22:49:33 557994105
Я белорус. Смогу ли стать космонавтом?
Аноним 12/04/20 Вск 22:52:21 557996106
>>557993
>Сможет ли в атмосфере венеры плавать подводная лодка?
Сомнительно:
>плотность воздуха у поверхности составляет 67 кг/м3, то есть 6,5 % от плотности жидкой воды на Земле[1].
С другой стороны, концептуально подводная лодка и дирижабль схожи...

>Будут ли работать вертолет, самолет?
Будут, почему бы нет. Но не земные, конечно. С подбором материалов будут трудности.
Аноним 12/04/20 Вск 22:54:24 557997107
>>557921
Спасибо за ответ, все благ тебе, анон
Аноним 12/04/20 Вск 22:57:54 557998108
>>557987
> А еще, даже потока нейтрино от взрыва (которые считай что ни с чем не взаимодействуют) достаточно, чтобы убить человека.

Каким образом? Что случится?
Аноним 12/04/20 Вск 23:01:06 557999109
>>557998
Ровно то же, что и с другими типами излучения: оно с чем-то взаимодействует, ломает всякие ДНК, белки и прочую хуйню. Поломок достаточно, чтобы организм не смог их починить и сломался.

В данном случае он там насчитал, что нейтрино настолько супермегаовердохуя, что даже при том, что с тушкой прореагирует одно из бразилиона, всего потока окажется достаточно, чтобы человека прибить.
Аноним 12/04/20 Вск 23:02:55 558000110
>>557994
Нет. Но не поэтому.
Аноним 12/04/20 Вск 23:17:42 558002111
Аноним 13/04/20 Пнд 00:29:10 558013112
>>557965
> Назовите хоть одну вменяемую причину, почему не производятся НИОКР клиновоздушных двигателей?
Так они проводились, и даже рабочие экземпляры были. Просто они говно. Дают небольшой выигрыш, а взамен горячие и тяжелые. Есть раздвижные сопловые насадки и дохуя других способов управлять степенью расширения в полёте, их и применяют.

Вроде ещё Firefly изначально хотели какой-то свой хитрый псевдо-клиновоздушник на первой ступени своей Alpha юзать, там рисовали 12 сопел и общая степень расширения регулировалась их параметрами, по сути то же самое что клиновоздушник получалось. Но с тех пор они поменяли всё 20 раз и хз что они там хотят теперь. В любом случае, пока ничего не запустили и не факт что вообще доживут до релиза, много таких по весне оттаяло.

>Зато профиты колоссальны — можно будет увеличить грузоподъёмность ракет в разы блять.
Нельзя. Это не волшебная технология будущего, может на несколько процентов импульс в среднем подымешь на всей траектории работы ступени и всё. Другие схемы проще и лишь немного менее эффективны.
Аноним 13/04/20 Пнд 00:43:39 558020113
>>558002
> The idea of neutrino radiation damage reinforces just how big supernovae are. If you observed a supernova from 1 AU away—and you somehow avoided being being incinerated, vaporized, and converted to some type of exotic plasma—even the flood of ghostly neutrinos would be dense enough to kill you.

Пиздец!
Аноним 13/04/20 Пнд 00:46:38 558021114
Если телепортировать чела из Австралии в Гренландию, то он упадет в низ головой?
Аноним 13/04/20 Пнд 00:47:06 558022115
Аноним 13/04/20 Пнд 00:53:53 558025116
>>558020
Вспотел от охуенности.
Аноним 13/04/20 Пнд 01:51:10 558032117
>>558021
Это противоречит теории плоской Земли.
Аноним 13/04/20 Пнд 02:43:19 558036118
если бы человеку не была нужна еда и пища, моэно было бы одной силой толкапния ногами прыгать между астероидами в поясе астероидов или по кольцу сатурна путешествуя и собирая данные? сила прыжка - достаточная тяга для больших астероидов?
Аноним 13/04/20 Пнд 02:52:28 558038119
>>558036
Если есть компьютер для рассчетов с вбитыми орбитами, а толкать ногами ты умеешь с филигранной точностью (чтобы не промахнуться мимо очередной цели), то пожалуй да. Большинство астероидов достаточно малы, чтобы с них можно было просто спрыгнуть. Но каждый прыжок будет охуенно долгим в среднем. Со скоростью 1 м/с, или сколько там можно достичь одним толчком, быстро не полетаешь, а астероиды не так чтобы тучами летают.
Аноним 13/04/20 Пнд 05:15:05 558044120
>>557965
>Назовите хоть одну вменяемую причину, почему не производятся НИОКР клиновоздушных двигателей?
Проводятся, только в другой области.

>Зато профиты колоссальны — можно будет увеличить грузоподъёмность ракет в разы блять.
Нет. Клиновоздушные сопла это не имба, а такой аналог регулируемого сопла Лаваля. С худшим кпд, худшим импульсом, но не требующий сложной механизации. Их основное применение - всякие боевые авиационные ракеты, которым надо уметь в многорежимность на всех высотах. Теоретически их можно воткнуть и просто на самолёт.

На космических ракетах они сильно дохуя профита не приносят (если мы не говорим о SSTO), ибо давая некоторый выигрыш в атмосферном УИ тяжёлый клин херит массовое совершенство, и что в итоге окажется выгоднее - ещё большой вопрос.

Вот если ёба скайлоны завезут - там да, всё совсем другими красками заиграет.
Аноним 13/04/20 Пнд 05:26:37 558045121
>>557909
А если не желтый карлан а какой-нибудь битлджус, у него по идее внутре можно летать же, нет?
Аноним 13/04/20 Пнд 05:36:49 558047122
>>557932
>Это возможно и применяется дохуя где.
Например? Вообще не представляю зачем может пригождаться левитирование мелких хуень.
Аноним 13/04/20 Пнд 05:41:34 558048123
>>558020
Мне нравится правило большого пальца для сверхновых.
"Если вы считаете, что вы насчитали какую-то цифру связанную со сверхновой - вы обсчитались, цифра будет выше"
Аноним 13/04/20 Пнд 05:43:44 558049124
>>557985
Дело не в стране, дело в продолжительности полета. На недельку слетаешь, без проблем выскочишь, год полетаешь - придется к земле обратно привыкать, самостоятельно только на четвереньках получится ползать.
Аноним 13/04/20 Пнд 05:46:57 558050125
image.png 1509Кб, 1280x720
1280x720
>>557994
Ну тащемта никто не запрещает.
Аноним 13/04/20 Пнд 06:43:20 558051126
>>558049
Ну это только если реально в невесомости нихуя не делать и ждать, пока ноги не отсохнут за ненадобностью.

Космонавты на МКС на тренажерах упражняются и бегают пристегнутыми к беговой дорожке, чтобы этого избежать, при возвращении на Землю они ходить более-менее могут.
Аноним 13/04/20 Пнд 10:14:37 558064127

Анончики, а есть ли инсайды, что ЧД уже рядом с нашей Солнечной вселенной?
Аноним 13/04/20 Пнд 10:15:11 558065128
>>558064
>вселенной
системой

Пытался избежать этого слова, но судьба решила иначе.
Аноним 13/04/20 Пнд 10:47:06 558067129
>>557985
Потому что американцы знают, что возвращаются в лучшую страну в мире, а русские возвращаются в сраную рашку
Аноним 13/04/20 Пнд 11:04:46 558068130
>>558064
какие инсайды в астрологии, долбоеб?
Аноним 13/04/20 Пнд 11:06:35 558069131
>>558068
Обычные, мистер мелкобуква.
Аноним 13/04/20 Пнд 11:27:03 558076132
Безымянный.png 25Кб, 800x800
800x800
Если, допустим в космосе, на планете Земля, вырыть яму (пикрелейтед), в половину толщи льда на речке/озере. Будет ли лед нарастать снизу (отмечено красным), ведь говорят иногда "речка промерзла до дна" Или лед тем толще, чем больше слежавшегося снега навалит на поверхность? как в Антарктиде
Аноним 13/04/20 Пнд 11:28:40 558078133
>>558076
>Будет ли лед нарастать снизу
Будет.
Аноним 13/04/20 Пнд 11:32:28 558079134
>>558051
Не, тренировки не панацея, кальций все равно вымывается. Тренировки просто откладывают слабость.
Аноним 13/04/20 Пнд 11:32:30 558080135
>>558078
А если продолжать рыть и ждать пока замерзнет, можно так до дна достать и построить там что-нибудь?
Аноним 13/04/20 Пнд 11:37:51 558082136
>>557985
Ты интерьер союза видел? Оттуда разве что кое-как вылезти на четвереньках можно. И то не факт что получится.
Аноним 13/04/20 Пнд 11:39:21 558083137
>>558080
Да, при условии, что выдержит давление.
Аноним 13/04/20 Пнд 11:41:01 558084138
Аноним 13/04/20 Пнд 11:48:25 558085139
f5b4c7017422409[...].jpg 345Кб, 1500x1000
1500x1000
>>558076
Это и на практике используется.
Аноним 13/04/20 Пнд 11:58:07 558086140
>>558085
Стрёмное зрелище.
Аноним 13/04/20 Пнд 12:39:05 558091141

Взрыв атомной бомбы в атмосфере это взрывная волна, излучение, распад некоторого количества атомов в газе, схлопывание образовавшейся разреженности.

А что есть такой взрыв в космосе? Количество превратившегося в газ урана недостаточно для мощной взрывной волны, остаётся только излучение.
Аноним 13/04/20 Пнд 13:22:36 558093142
>>558085
Это тот самый самосбор?
Аноним 13/04/20 Пнд 13:44:33 558095143
>>558085
Спасибо анон, нагуглил фото, почитал и понял что затея бессмысленная. Это в Якутии при -50 имело бы смысл, а мне похоже придется таки менять русло реки, как изначально задумывал.
Аноним 13/04/20 Пнд 14:26:05 558099144
>>558050
Но у нас нет своего космического агенства. В Роскосмосе написано только граждане России, а на сайте европейского агенства что берут только граждан членов. И вообще, последний набор у них был в 2009
Аноним 13/04/20 Пнд 15:08:11 558105145
>>558091
Свет, энергия, рандомные атомы и молекулы и немножко радиации
Аноним 13/04/20 Пнд 16:07:20 558117146
А куда исчезает энергия? А она исчезает?
Аноним 13/04/20 Пнд 16:08:08 558119147
Стикер 255Кб, 500x500
500x500
>>558099
К китайцам просись.
Аноним 13/04/20 Пнд 16:27:01 558124148
>>558119
Так у них всё через раз взрывается, а мне бы ещё пожить хочется. Думаю паспорт европейский получить и пробовать в Европе. Тем более их точно пустят в американские лунные спутники.
Аноним 13/04/20 Пнд 16:31:09 558126149
>>558124
Чейта странно что все эти интеркосмосы того, раньше был вполне валидный способ полетать не имея космической программы в стране.
Аноним 13/04/20 Пнд 16:47:11 558128150
>>558126
Это ведь был чисто политический процесс. Политики не стало, не стало и интеркосмоса.
Аноним 13/04/20 Пнд 17:00:43 558130151
Аноним 13/04/20 Пнд 22:58:12 558202152
А почему молния синяя? Из за кислорода? В вакууме молния может быть? Какого там она цвета? Что с позитронами в вакууме? Позитронная молния будет аннигилироватся в атмосфере? Каков цвет данного типа аннигиляции? Ультра-гамма или будет кусочек видимого света? Будет ли эта позитронная молния прямой или она аннигилирует у источника и превратится в шар?
Аноним 13/04/20 Пнд 23:07:58 558207153
>>558202
Молния не обязательно синяя, это сильно зависит от температуры и состава атмосферы, разные типы выглядят по-разному.
В вакууме быть не может, ибо светится плазма, образованная из газа нагревом, вызванным протекающим по каналу разряда током. Нет газа - нечему светиться. Но в космосе полно летающих заряженных частиц, особенно в радиационных поясах, поэтому в спутники ставят разрядники, чтобы между корпусом и потрохами разряд был контролируемый, а не абы какой.

Да, позитроны аннигилируются сразу. Цвет никакой, там гамма образуется.
Аноним 13/04/20 Пнд 23:15:34 558212154
>>558202
>А почему молния синяя?
Вблизи белая, но издалека иногда может видеться синей из-за атмосферы.
>В вакууме молния может быть?
В вакууме нет, так как проводника не будет.
> Что с позитронами в вакууме?
Ничего, пока не встретят электроны.
>Каков цвет данного типа аннигиляции? Ультра-гамма или будет кусочек видимого света?
Во всём спектре.
>Будет ли эта позитронная молния прямой или она аннигилирует у источника и превратится в шар?
Я не очень представляю как ты собираешься получить позитронную молнию в земных условиях, все позитроны в атмосфере довольно быстро аннигилируются, так что кина не будет, и к тому же, как позитроны с зарядом + сделают молнию в земной атмосфере, не ясно отрицательные ионы (???).
Аноним 13/04/20 Пнд 23:18:33 558215155
>>558212
>>Каков цвет данного типа аннигиляции? Ультра-гамма или будет кусочек видимого света?
>Во всём спектре.
Ето точно? Аннигиляция же вроде совсем не как черное тело излучает.
Аноним 13/04/20 Пнд 23:22:53 558218156
>>558212
аннигиляция e+ и e- порождает два фотон с заведомо известной энергией, это узкая полоска в спектре
Аноним 13/04/20 Пнд 23:23:12 558219157
>>558218
>два фотон
и адын булка
Аноним 13/04/20 Пнд 23:39:16 558230158
>>558215
>>558218
Ну по идее же ета хуйня должна выделять много энергии, которая будет превращаться в охуительно яркий свет, ну то есть в вакууме так, в атмосфере просто нагреет окружающее вещ-во и оно тоже будет светиться может где и проебался, незнаю
Аноним 13/04/20 Пнд 23:48:48 558236159
У меня получилось длинна волны анигиляционного электрон-позитронного фотона 2.437 пикометра. Я правильно посчитал?
Аноним 13/04/20 Пнд 23:56:41 558238160
>>558236
я понимаю что это тред тупых вопросов, но там формула-то в куркулятор засунуть и 2 кнопки нажать
Аноним 14/04/20 Втр 00:05:42 558242161
>>558230
Яркость света зависит от количества фотонов, а не от их энергии. При аннигиляции, как уже написали, получается всего 2 или 3 высокоэнергетических фотона, так что никакого яркого света не будет, будут только гамма-лучи.
Аноним 14/04/20 Втр 00:07:20 558243162
1586812035473.jpeg 90Кб, 640x472
640x472
1586812035495.jpeg 411Кб, 1455x2204
1455x2204
ЧТО ЭТО ЗА ХУЙНЯ БЛЯТЬ

80 тонн дури, что совки там запускали?
Спутник нахуй, 80 тонн ебать.
Этой хуйней собрались города сжигать?
Что у нее внутри?

Какой самый тяжёлый когда-либо запущенный спутник?
Аноним 14/04/20 Втр 00:09:39 558244163
>>558243
Это макет по большей части, лазера там не было.
Аноним 14/04/20 Втр 00:13:41 558246164
264.jpg 52Кб, 571x304
571x304
Ху из це? И почему мне это кажется абсурдным?
Аноним 14/04/20 Втр 00:13:55 558247165
>>558243
Внутри был чугуний, движки и несколько экспериментов.
Города таким не посжигаешь, а вот другие спутники из строя повыводить можно.
Самый тяжелый спутник одним куском, без учета сборки на орбите - Skylab. (т.к. эту хуиболу не вывели)
Аноним 14/04/20 Втр 00:14:37 558248166
>>558246
Больше сжимаем - выше плотность, в чем абсурд?
Аноним 14/04/20 Втр 00:14:39 558249167
>>558247
Скайлаб не спутник
Аноним 14/04/20 Втр 00:15:11 558250168
>>558249
Вполне себе искусственный спутник Земли, еще и обитаемый.
Аноним 14/04/20 Втр 00:18:00 558251169
>>558246
Все что между НЗ и ЧД - лишь предсказано, не обнаружено
Аноним 14/04/20 Втр 00:22:42 558252170
>>558248
Разве не окукляться они под горизонт?
Аноним 14/04/20 Втр 00:22:52 558253171
>>558251
Мне казалось, НЗ уже вот почти, типа скорость убегания в районе трети световой. Неужели там еще есть столько места для дальнейшего уплотнения?
Аноним 14/04/20 Втр 00:24:26 558254172
>>558251
Всякие преоны предсказаны только шизиками, никакого реального обоснования у них нет.
Аноним 14/04/20 Втр 00:29:56 558255173
>>558253
>>558252
Места там вплоть до шварцнеггера, а НЗ больше его в разы. Размеры в диаграмме может и абсурдны, да. И шварцнеггеровская плотность это что-то типа 10^26, так что и преонные звезды там не сходятся

>>558254
Верно.
Аноним 14/04/20 Втр 00:32:47 558256174
>>558253
Объем падает согласно кубу от радиуса, если ты об этом (и плотность растет)
Аноним 14/04/20 Втр 00:35:37 558257175
>>558256
Не совсем. Насколько я помню, НЗ больше радиуса Шварцшильда в несколько раз (зависит от массы), но не порядки. А в посте для "преонной звезды" какие-то размеры, которые мне кажутся совершенно нереальными.
Аноним 14/04/20 Втр 00:37:52 558258176
>>558255
Шварцнейгер неплохо так поднасрал Шварцшильду.
Аноним 14/04/20 Втр 00:38:14 558259177
>>558257
Да собственно для Солнца радиус Шварцшильда в районе 3 км.
Аноним 14/04/20 Втр 02:14:16 558269178
image.png 1502Кб, 1280x720
1280x720
image.png 103Кб, 300x300
300x300
image.png 760Кб, 1000x1000
1000x1000
>>558207
>В вакууме быть не может, ибо светится плазма
>>558212
>В вакууме нет, так как проводника не будет.
А почему в разреженном газе светится лучше, чем в н.у.?
Аноним 14/04/20 Втр 02:15:30 558271179
>>558243
Мне до сих пор пичот что они его вверх тормашками запускали.
Аноним 14/04/20 Втр 08:57:01 558283180
почему нельзя утилизировать ядерные отходы в вулкане? они же тяжелее камня и утонут в самый низ
Аноним 14/04/20 Втр 10:27:08 558285181
С помощью каких методов хотят обнаруживать внеземную жизнь?
Свободный кислород в атмосфере достаточный показатель?
Аноним 14/04/20 Втр 10:47:39 558286182
>>558283
>утилизировать ядерные отходы в вулкане
Шикарный способ:)
Аноним 14/04/20 Втр 10:49:59 558288183
>>558286
ну да, они утонут в центр и никогда больше не будут проблемой
Аноним 14/04/20 Втр 11:12:56 558289184
>>558288
Лучше на солнце ракетой запускать
Аноним 14/04/20 Втр 11:16:01 558290185
>>558289
это очень энергетически невыгодно и если ракета ебнет, то говно радиоактивное по космодрому раскидает
нафиг ты толстишь?
Аноним 14/04/20 Втр 11:17:06 558292186
>>558289
Выбросить их из Солнечной Системы гораздо проще, чем уронить на Солнце.
Аноним 14/04/20 Втр 11:22:56 558293187
>>558292
не пизди, на солнце проще если через юпитер лететь
да можно вообще прямо на юпитер, хули ему будет
Аноним 14/04/20 Втр 11:31:16 558294188
>>558293
В целом, если через Юпитер, то уже похуй куда.
Аноним 14/04/20 Втр 11:36:02 558295189
>>558293
Юпитерцы будут против.
Аноним 14/04/20 Втр 11:48:28 558297190
>>558285
Недостаточный. Но шансы повышают. Пока что есть только такие методы маркировочных газов в атмосфере. Ещё ищут сферу дайсона. Зачем-то. Пока дальний космос никак больше не исследуют. Нету методов.
>>558283
Это для тебя они отходы, а для людей будущего может это бесценный ресурс за которые будут вестись войны.
Если ты кинешь объект Х в вулкан. Он не утонет пропутешествовав к ядру планеты. Во первых вулкан это тебе не мантия, а расплавленная земная кора, которая по сложным тоннелям нагретая от верхней мантии, поднимается по закону архимеда наверх. И как ты заставишь утонуть этот ваш урановый лом, если происходит извержение и там как бы поток вверх. Двигатель поставишь и поплывёшь? По вашему отходы атомного производства тяжелее чем порода или это вообще чистый уран. А если это тритиевая вода? Насколько я знаю отходы эти сильно разбавлены дабы не облучится при транспортировке. И они скорее всплывут, а не утонут. И последнее, по повожу вулканов. Вулкан это тебе не водичка это, блять, тысячи градусов расплавляющие и испаряющие эти ваши отходы. Кидает, значит, вася мешок урана в вулкан и получает рак лёгких от надышавшихся паров.
Про солнце уже ответили. Опасно, не эффективно,дороже чем выкинуть из системы.
А Юпитер сука не трогай. У нас за такой на спейсаче убивают нахер.
Аноним 14/04/20 Втр 11:52:37 558300191
>>558297
>как ты заставишь утонуть этот ваш урановый лом
По закону Архимеда, блядь
Аноним 14/04/20 Втр 11:53:01 558302192
А нахуя эти отходы вообще куда-то далеко девать? Может, смешать с породой и засыпать обратно в шахты? Самые злые элементы распадутся в течении пары лет, через несколько десятков состав будет близок к естественному. Где я наебался?
Аноним 14/04/20 Втр 11:56:50 558304193
>>558302
Вообще большинство "ядерных отходов" которые в энергетических реакторах вполне перерабатываемые и дохуя из них можно использовать дальше.
Только дешевле новый ураниум прогнать через обогатительный, а не перерабатывать ОЯТ и переиначивать режим работы АЭС
Аноним 14/04/20 Втр 12:12:03 558313194
>>558300
Расплавится за 5 минут и затем всплывёт. Если кусок большой ударится о повёрнутую на 90 градусов стенку жерла. и застынет на глубине 10-100 метров А если извержение продолжится, поток вымоет его наверх и он не утонет, будет барахтатся на поверхности.
Аноним 14/04/20 Втр 12:22:36 558315195
>>558313
Есть же просто открытые жерла вулканов где просто лава плещется, не вытекая никуда на скоростях, туда надо скидывать.
Аноним 14/04/20 Втр 12:37:43 558317196
>>558294
до юпитера меньше дельты чем до солнца или за пределы солнечной системы
Аноним 14/04/20 Втр 12:39:57 558318197
>>558317
Давайте не будем засирать Юпитер, нам еще с него газики собирать через три тысячи лет.
Аноним 14/04/20 Втр 12:45:05 558319198
>>558318
лучше с сатурна, там гравитация как на земле и красивые спутники/кольца
на юпитере ниче нет
Аноним 14/04/20 Втр 12:46:45 558320199
>>558319
Я потому и сказал, через 3000, сатпрррн уже выкачали, оставили хтоническую планету для колонизации.
Аноним 14/04/20 Втр 12:50:53 558321200
>>558117
Вся вселенная это безконечный цикл перехода энергии из одного состояния в другое.
Аноним 14/04/20 Втр 12:56:40 558323201
>>558321
про тепловую смэрт забыл, шиз?
Аноним 14/04/20 Втр 12:57:40 558324202
>>558246
Вдогонку к вопросам этого анона вопросы.

Слыхал истории шокирующие про ЧД с диаметром в 1 000 000 000 км и плотностью как у газа, ну маленькой кароч.

Такое возможно? Не может ли быть так, что такие объекты растут, а потом просто разваливаются и превращаются в газ там или ещё во что-нибудь?

Третий.

Может ли быть некая предельная кинетическая энергия для скажем протона( нейтрона ), или температура, при которой он распадатеся?

Это важно, ведь следующий шаг это бомба на таком вот процессе, Солнечная система Вселенная в труху.
Аноним 14/04/20 Втр 12:58:28 558325203
>>558323
Понятное дело, что для бобразоанных это дико прозвучит, но я думаю, что ты и сам понимаешь, что это не так.
Аноним 14/04/20 Втр 13:01:50 558327204
>>558325
что не так, я не понимаю о чем ты
Аноним 14/04/20 Втр 13:03:00 558328205
>>558327
Утверждать возможность тепловой смерти вселенной это как в начале 20 века доказывать, что Солнце это диск, что по куполу наверху ползает.
Аноним 14/04/20 Втр 13:03:31 558329206
>>558324
Йеп, СМЧД имеют среднюю плотность как говно.
Только средняя плотность не отменяет горизонт событий и массивность.
Напихай немного пердежа в пределы орбиты сатурна чтоб был легкий душок - этой массы будет достаточно чтобы ЧД была в таком объеме.
СМЧД живут охуительно долго, по Хокингу быстрей всего разваливаются легкие дыры.
Аноним 14/04/20 Втр 13:04:25 558330207
>>558328
В зогач, животное, вполне валидная гипотеза.
Если у тебя есть реальные аргументы против нее - приводи их, а не свои мантры-аналогии из зогача приводи.
Аноним 14/04/20 Втр 13:05:06 558331208
>>558328
ннно тепловое равновесие и все такое
Аноним 14/04/20 Втр 13:06:36 558332209
>>558329
Так же мне интересно.

А если её дестабилизировать? Взрыв там сверхновой? Просто интуитивно понятно, что эт нечно должно развалиться.

По моему не я один такой умный и даже читал про это, мол ЧД появляется, растёт до некоторого предела, разрывается, её остатки собираются в газ, газ в звёзды, ну и далее по циклу.
Аноним 14/04/20 Втр 13:08:12 558333210
>>558330
>Если у тебя есть реальные аргументы против нее

Изи - МЫ ДО СИХ ПОР ЖИВЫ, под мы не только мы, но и вселенная в целом.

>>558331
Не вижу никаких проблем. Меня просто шокирует такая категоричность в утверждениях и это на фоне прошлых ошибок. Причём когда-то даже ртутью или кровопускание лечили, наверняка ещё и докторские защищали.
Аноним 14/04/20 Втр 13:20:10 558334211
>>558332
>Просто интуитивно понятно, что эт нечно должно развалиться.
Нет. Это реально ЧД.
У горизонта событий скорость убегания выше скорости света. Никто никуда не сваливает.
> ЧД появляется, растёт до некоторого предела, разрывается, её остатки собираются в газ, газ в звёзды, ну и далее по циклу
Таких гипотез всерьез не рассматривают.
Интуиция - очень вредная вещь в науке. Не используй её.
Аноним 14/04/20 Втр 13:21:00 558335212
>>558333
>Изи - МЫ ДО СИХ ПОР ЖИВЫ, под мы не только мы, но и вселенная в целом.
Так че, епт, охуенная логика.
Я бессмертен получаюсь.
Я ж до сих пор не умер. Значит смерти для меня не существует.
Гениально.
Аноним 14/04/20 Втр 13:25:58 558336213
>>558334
Не согласен, новые явления открываются на на бумаге, а в эксперименте, в котором она очень нужна и важна.
Аноним 14/04/20 Втр 13:27:26 558337214
>>558335
Ты точно из учёных? Ты глупость пишешь как доказательство, а из доводов у тебя есть гипотезы учёных, которые очевидно не сбудутся, я не просто так привёл в пример прошлые ошибки, это печально, если реальные учёные современности так же глупо наступают на те же грабли.
Аноним 14/04/20 Втр 13:31:36 558338215
>>558337
Ещё раз - гипотезу развенчать можешь нормально? Делай. Не можешь - помалкивай и не позорься.
Тепловая смерть не считается несбыточной фантазией и рассматривается всерьёз.
Аноним 14/04/20 Втр 13:32:28 558339216
>>558336
Ни искажения п-в, ни квантомех не подвергаются интуиции, что ты несешь, не позорься.
Аноним 14/04/20 Втр 13:33:24 558340217

>>558338
>гипотезу

Ахахахаха.

Кода там гравицапу сделают, м? А то столько визгу про стройную теорию гравитации, лол.

Ты бы сам лучше промолчал, чем эту дурь защищать.
Аноним 14/04/20 Втр 13:34:09 558341218
>>558339
А математика и физика вполне, я так в вузе их сдавал за 30 и 10 минут.
Аноним 14/04/20 Втр 13:39:42 558342219
>>558340
В зогач, быдло. Или в Б. Где твои быдлобратья живут.
Аноним 14/04/20 Втр 13:40:10 558343220
>>558341
А квантомех и макромир нет.
Аноним 14/04/20 Втр 13:42:29 558344221
>>558343
Потому что не все законы известны и смотря как на все результаты смотреть.
Аноним 14/04/20 Втр 13:42:50 558345222
>>558344
Нет, они неинтуитивны.
Аноним 14/04/20 Втр 13:42:56 558346223
>>558342
Агрессивный учёный, позор.
Аноним 14/04/20 Втр 13:43:19 558347224
>>558345
Ну приведи пример не интуитивного.
Аноним 14/04/20 Втр 13:44:45 558348225
>>558346
Пошёл на хуй.

>>558347
КВАНТОВАЯ МЕХАНИКА БЛЯДЬ

Пиздец, откуда такая имбецилия в треде? Куда моча смотрит!?
Аноним 14/04/20 Втр 13:46:54 558349226
>>558348
Вот именно, почему вместо адекватного ответа я получаю какую-то истерику?

Ты можешь адекватно ответить на вопрос или нет?

И да, что твой квант мех позволил создать за последние 30 лет, удиви меня, он ведь весь такой верный и всё привсё предсказывает.
Аноним 14/04/20 Втр 13:49:25 558350227
>>558349
Сука, иди на хуй, говно ебаное.
Я нормально пытаюсь помогать, поддерживаю тред, и ты своим жиром меня выбешиваешь, скотина ебаная.
ИДИ НА ХУЙ И НЕ ВОЗВРАЩАЙСЯ ПИДОР ТУПОРЫЛЫЙ
НЕ ЕБИ МОЗГИ ГНОЙ
Аноним 14/04/20 Втр 13:49:59 558351228
>>558350
>>558349
Хотя нет, сгний в аду, я больше сюда не захожу, чтоб таких пидорасов не видеть как ты, охуела параша ебаная, мозги ебать блядина хуесосная
Аноним 14/04/20 Втр 13:52:22 558352229
Аноним 14/04/20 Втр 14:02:15 558353230
>>558352
Я моделировал системы, которые стремятся к некому равновесию, да, это мгновенно схлопываются, но как только я предположил, что во вселенной есть замкнутый цикл перехода состояний вещества, то вселенная более не схлопывалась.
Аноним 14/04/20 Втр 14:08:42 558355231
>>558324
Не стоит понимать СМЧД как кисель. У неё всё так же вроде как бесконечная плотность в сингулярности и отсутствие вещества вплоть до горизонта. В среднем да кисель, но это средняя температура по больнице. Та же плотность населения 0.00001 человек на квадратный километр, не означает, что человека порубили и раскидали по тайге.
Аноним 14/04/20 Втр 14:15:22 558356232
>>558353
Ты это блядь к чему? Никаких замкнутых циклов не нужно, если только ты не хочешь вечную вселенную.
Аноним 14/04/20 Втр 14:22:54 558359233
>>558356
Я это блядь к тому, что всё схлопывается, если есть некое центральное состояние или нечто стремится усреднить разницу между чем-то.

Ты как вообще себе жизнь вселенной представляешь? Ты серьёзно что ли веришь, что вот это всё однажды как выше ждёт тепловая смерть или оно сколлапсирует в огромную ЧД?

Это как если бы появилась теория про то, что лес скоро умрёт или сольётся в одно дерево, или кучку золы.
Аноним 14/04/20 Втр 14:32:50 558360234
>>558359
>Я это блядь к тому, что всё схлопывается, если есть некое центральное состояние или нечто стремится усреднить разницу между чем-то.
Что блядь схлопывается, ты можешь нормальным языком говорить?

>Ты серьёзно что ли веришь, что вот это всё однажды как выше ждёт тепловая смерть
Да, ну по крайней мере допускаю, почему нет-то? И что это вообще за аргумент такой?

>Это как если бы появилась теория про то, что лес скоро умрёт
Любой лес обязательно умрёт.
Аноним 14/04/20 Втр 14:44:55 558362235
>>558333
че ты сказать хочешь вообще?
то есть если наука раньше ошибалась в чем-либо, то значит можно с полной уверенностью заявлять что она ошибается и сейчас, а доказательства не нужны?
Аноним 14/04/20 Втр 14:50:54 558365236
>>558349
Как минимум туннельный диод.
Аноним 14/04/20 Втр 14:53:33 558366237
Ну вообще, тепловая смерть всего лишь одна из гипотез. Но тот анон и правда даун, с криками "НИ МОЖЫД И ВСЁ ЕТО ВАМ ЖИДОМАСОНЫ ПОДМЕНИЛИ НОУКУ ТОЛЬКО В КАМЕННЫЙ ВЕК".
Аноним 14/04/20 Втр 16:39:55 558373238
>>558290
>если ракета ебнет
А мы на лифте.
Аноним 14/04/20 Втр 17:06:30 558378239
>>558247
Тогда уж шаттл с нагрузкой
Аноним 14/04/20 Втр 17:45:34 558383240
>>558317
На гравиманевр у Юпитера дельты не нужно вообще. Так что без разницы.
Аноним 14/04/20 Втр 17:46:26 558384241
>>558247
А третья ступень сатурна с аполлоном не тяжелее?
Аноним 14/04/20 Втр 18:25:18 558390242
>>558250
Речь про беспилотный КК без СЖО
Аноним 14/04/20 Втр 18:29:23 558391243
>>558318
Думаю, если на Юпитер всю Землю уронить, то он не сильно расстроится
Аноним 14/04/20 Втр 18:41:07 558392244
Аноним 14/04/20 Втр 18:46:18 558393245
1586879174340.jpeg 6Кб, 300x144
300x144
>>558347
Двухщелевой опыт с одиночными фотонами даёт интерференционную картину, хотя это неинтуитивно
Аноним 14/04/20 Втр 19:11:37 558398246
15867750884352.jpg 111Кб, 1042x977
1042x977
Про горизонт событий чёрной дыры вопрос.
Допустим, я лечу на продвинутом корабле и ныряю под поверхность горизонта событий, настолько чуть-чуть, что там 0,(0)1% будет, чтобы метафорически высунь ты руку из форточки, и она вылезет за горизонт событий наружу. Так как горизонт событий - это ещё не сама чёрная дыра, как я понял, а всего лишь та область откуда фотоны обратно уже не улетают назад, но и не значит, что непосредственно падают на математическое тело ЧД, а значит, часть фотонов, а следовательно и энергии, тупо вращается в вечном орбитальном круге, как планеты вокруг своей звезды столь же стабильно крутятся на протяжении всех тех столетий, как человек изобрёл телескоп, и никогда не упадут на ЧД, но и не вылетят за горизонт. Следовательно, как я думаю, между самой ЧД и внутренней поверхностью горизонта событий обязана быть поле вращения всех частиц от весьма до бесконечно наполненного энергие. Следовательно, если вращаться по направлению движения вращения ЧД под горизонтом, когда энергия дует тебе в сраку корабля, её можно собирать и собирать, а потом, на очередном витке, использовать всё накопившееся для рывка внхуйъ (не вверх, перпендикулярно касательной к поверхности прочь от ядра самой ЧД, а просто чуть с большим нажимом вперёд, чтобы обзавестись орбитой выходящей за орбиту горизонта событий ЧД). И уже выйдя за пределы горизонта, съебать как из обычной внечёрнодырной области, с полными баками и карманами фотонов, глюонов или кварков - любой количественно измеримой порции материи - и ебашить в космос дальше.
Итак, что думаешь? Насколкьо такая теория может оказаться правдивой?
Аноним 14/04/20 Втр 19:24:09 558401247
>>558398
Чтобы съебать из-под горизонта событий, нужно двигаться быстрее скорости света. Как только представишь способ это сделать - пожалуйста, летай туда, обратно, сколько хочешь.

На самом деле из-под горизонта событий можно туннелировать. Но это не точно.
Аноним 14/04/20 Втр 19:39:48 558402248
>>558398
Ни на сколько. Во-первых, фотонная сфера находится на расстоянии 1.5 Шварцнеггеровских радиусов, ниже неё никаких стабильных орбит нет даже для безмассовых тел.

Во-вторых, фотонная сфера неустойчива, фотоны там не движутся по эллипсам как планеты. Планеты не улетают со своих орбит потому, что удаляясь от Солнца, они теряют скорость и начинают падать обратно, но фотоны всегда движутся со скоростью c, законы Кеплера для них не работают.

Если фотон окажется выше радиуса фотонной сферы хотя бы на нанометр, он начнет удаляться от черной дыры по раскручивающейся спирали и рано или поздно вылетит в бесконечность, и наоборот — если он окажется хоть чуть-чуть ниже этого радиуса, у него дорога только под горизонт.

В-третьих, для высасывания энергии из вращающейся черной дыры никакие фотоны нахуй не нужны, можно извлекать энергию просто из вращающегося пространства-времени, которое черная дыра тянет за собой.

https://en.wikipedia.org/wiki/Penrose_process
Аноним 14/04/20 Втр 19:47:00 558404249
>>558398
Даже имея бесконечную энергию, из-под горизонта событий не вырваться.

Ты пойми, пространство изогнуто блять, как если бы ты находился на поверхности мяча, но внутри него.
Как быстро ни двигайся, за его пределы тебе не вырваться.
Аноним 14/04/20 Втр 19:48:37 558406250
>>558402
>Если фотон окажется выше радиуса фотонной сферы хотя бы на нанометр, он начнет удаляться от черной дыры по раскручивающейся спирали и рано или поздно вылетит в бесконечность, и наоборот — если он окажется хоть чуть-чуть ниже этого радиуса, у него дорога только под горизонт.
Но ведь так и получается, ведь это излучение Хоккинг - частица0 попадая на границу горизонта событий распадается на частицу+ и частицу-, одна из которых съёбывается прочь, другая остаётся в пределах ЧД.
Аноним 14/04/20 Втр 19:55:21 558408251
>>558404
а если на альбукерке?
Аноним 14/04/20 Втр 19:57:25 558409252
Но ведь пространство-время не является гладким, на микроуровне оно кипит и пенится, и на горизонте чд тоже.
А значит, можно подлететь к горизонту и в щёлочку раз раз и пролезть наружу.
Как сквозь текстуры в игре.
Аноним 14/04/20 Втр 20:05:46 558410253
Я не понял, нефть это что ли сгнившие древние животные?
Аноним 14/04/20 Втр 20:09:46 558411254
>>558410
Да, животные и растения, как и каменный уголь. Вообще это не по теме космоса. не находишь?
Аноним 14/04/20 Втр 20:10:23 558412255
>>558393

Только если ты фотон представляешь как кусочек синусоиды летящий куда-то.

А вот если волна...
Аноним 14/04/20 Втр 21:11:01 558416256
>>558411
Я просто задумался, как так образовывались озёра с нефтью, это же надо нереальное кол-во биомассы чтобы в одно место складывалось. И почему одно становится камнем, а другое нефтью? Я не понимаю.
Аноним 14/04/20 Втр 21:11:57 558417257
>>558393
А если такой опыт с одним электроном провести?
Аноним 14/04/20 Втр 21:12:17 558418258
>>558416
нефть идет с тех времен когда деревья были по количеству как трава, а по размерам в разы больше
Аноним 14/04/20 Втр 21:14:59 558419259
>>558416
>>558418
Есть теории абиогенного происхождения нефти, если что.
Аноним 14/04/20 Втр 21:18:22 558420260
>>558416
>>558418
Дело не в количестве и размере деревьев даже, а в том, что они тупо не разлагались почти — в каменноугольном периоде грибы и бактерии еще не научились эффективно жрать древесину.

Сейчас такое происходит только в болотах, где биомасса быстро тонет без доступа воздуха, а тогда такое по всей поверхности планеты происходило, деревья не гнили, а как есть в землю и погружались. В результате под землей были захоронены колоссальные объемы углерода, которые мы сейчас потихоньку в атмосферу и выпускаем.
Аноним 14/04/20 Втр 21:20:09 558421261
>>558420
Т.е. это в основном деревья, а не трупы динозоидов?
И еще, почему некоторые деревья превращалиь в уголь, а некоторые в нефть?
Аноним 14/04/20 Втр 21:21:06 558422262
15742401140710.jpg 33Кб, 554x235
554x235
15742401140711.jpg 75Кб, 324x813
324x813
>>558417
Уже проводили в самом что ни на есть натуральном виде. Две щели, сдвижной экран, который мог закрывать одну щель или даже обе (для контроля), электронная пушка, выпускающая 1 (прописью — один) электрон в секунду и детекторный экран, который в реальном времени мог показывать столкновения.

https://physicsworld.com/a/feynmans-double-slit-experiment-gets-a-makeover/ (есть видео)
https://iopscience.iop.org/article/10.1088/1367-2630/15/3/033018

Одна щель — распределение равномерное, две щели — классическая дифракционная картина, хотя электрон был в каждом пролете всего один и интерферировать ему было тупо не с чем.
Аноним 14/04/20 Втр 21:34:54 558423263
>>558422
>интерферировать ему было тупо не с чем.
А с самим собой? Это же волна. Копроскулярно-волновой дуализм.
Аноним 14/04/20 Втр 21:39:06 558425264
>>558421
Нефть образовалась абиотическим путем. Не слушай поехавшего.
Аноним 14/04/20 Втр 21:41:08 558426265
>>558393
>контринтуитивно
Только если ты не представляешь пространство как совокупность плотноприжатых сфер.
Аноним 14/04/20 Втр 21:41:58 558427266
>>558425
>Не слушай поехавшего.
Хороший совет, зря ты в своё время его не придерживался, а теперь иди нахуй. Биогенная теория главенствующая, и не просто так.
Аноним 14/04/20 Втр 21:44:20 558429267
Аноним 14/04/20 Втр 21:47:21 558430268
Согласно положениям СТО, не существует некоей выделенной системы отсчета, все инерциальные СО равноправны.

Мне вот что интересно. Возьмем достаточно большой объем пространства и вычислим центр масс содержащегося в нем вещества. На больших масштабах, согласно наблюдениям, Вселенная более-менее однородна. Поэтому, центр масс будет двигаться более-менее по инерции (со всех сторон "примерно одинаково" вещества, так что его никуда не тянет), то есть с ним можно связать систему отсчета. Если неограниченно увеличивать этот объем, можно построить такую СО, относительно которой вещество в среднем неподвижно. Так вот, такая СО будет чем-то интересна? Или это чисто умозрительная хуйня, ни на что не влияющая?
Аноним 14/04/20 Втр 21:47:33 558431269
Аноним 14/04/20 Втр 21:58:55 558432270
>>558430
>Согласно положениям СТО
Это выдумка евреев
Аноним 14/04/20 Втр 22:00:12 558433271
>>558430
Ну, наверное будет интересна тем, что она ближе всего к действительно инерциальной.
Аноним 14/04/20 Втр 22:05:24 558435272
>>558430
Такая система придумана давным-давно, скорость считается относительно реликтового микроволнового фона, который равномерно всю Вселенную заполняет. Применений у нее по факту все равно почти никаких.
Аноним 14/04/20 Втр 22:09:06 558438273
>>558435
>Такая система придумана давным-давно
Ну я и не особо рассчитывал заработать нобелевку постом на харкаче.
>скорость считается относительно реликтового микроволнового фона, который равномерно всю Вселенную заполняет
Не уверен, что понимаю. Имеется в виду, что точка покоится в этой СО, если фон имеет одинаковую температуру со всех сторон?
Аноним 14/04/20 Втр 22:15:14 558439274
md53s1111b.jpg 78Кб, 612x792
612x792
>>558438
>Имеется в виду, что точка покоится в этой СО, если фон имеет одинаковую температуру со всех сторон?

Как-то так, да, и по красному и синему смещению фона можно выяснить, куда и с какой скоростью движется тело. Земля в данный момент относительно него движется на скорости в примерно 370 км/с.
Аноним 14/04/20 Втр 22:17:51 558441275
Аноним 15/04/20 Срд 00:12:32 558455276
Аноним 15/04/20 Срд 01:06:26 558458277
>>558455
А он и теряет. Но медленно. Или ты думаешь, что пыль и лёд в кольце дохуя тяжёлые?
Аноним 15/04/20 Срд 01:08:29 558459278
>>558455
Не теряет. Суммарно энергия остаётся та же. По той же причине волны и прочие фичи в кольце нихуя не замедляют. Вот такой парадох.
Аноним 15/04/20 Срд 08:48:24 558467279
>>558419
Им похуй. Из диназаврав и точка!
Аноним 15/04/20 Срд 10:39:46 558471280
>>558467
Ну приведи аргументы в пользу абиогенного происхождения.
Аноним 15/04/20 Срд 10:43:17 558472281
Как называется явление, когда два тела и мелкое все рано как-то изучают видятза счёт того, что масса первого выше.
Аноним 15/04/20 Срд 16:18:42 558506282
>>558472
Транзит?
Линзирование?
Аноним 15/04/20 Срд 17:18:44 558508283
Если ЧД может быть любой плотности, то возможно что мы живём внутри гигамассивной ЧД?
Аноним 15/04/20 Срд 18:03:10 558512284
>>558508
Тебе ли не похуй?

В смысле, это ничего не меняет. Мы наружу никак не выберемся, и что там снаружи - никак не узнаем, принципиально. То же самое что и утверждение, что нет никакого "снаружи" и никогда не было.

У кого-то были даже манягипотезы про "естественный отбор" вселенных из чёрных дыр, такое там.

Аноним 15/04/20 Срд 18:12:02 558513285
>>558512
Кстати, а есть хоть какие-то теории насчет настолько ебически огромных дырков? Это ведь даже не галактические СМЧД, тут все намного хуже. Ведь гравитация, как и все остальное, вроде как распространяется не быстрее света. Что будет, если гравитации миллиард лет только "ползти к краю дыры"? Расширение пространства, опять же. Что если край дыры за счет этого расширения улетает от ее центра быстрее скорости света?
Аноним 15/04/20 Срд 19:09:45 558520286
Аноним 15/04/20 Срд 19:42:11 558525287
Аноним 15/04/20 Срд 20:08:26 558531288
>>558513
Я не тру-астрофизик, но насколько хватает понимания - у ЧД нет теоретического предела массы/размера, и кроме излучения Хокинга не известно ни одного процесса способного обратить её рост не может. Что знаю достоверно - это то, что во вселенной есть большие ЧД, есть очень большие, и в астрономическом смысле нет никакой разницы сколько гравитации "ползти" - несколько часов или несколько миллионов лет, она всё равно оказывает своё влияние.

Строго говоря ЧД это не совсем объект, а область пространства вокруг объекта содержащего всю её массу - сингулярности. И "край", про который ты говоришь - горизонт событий - это не физический объект в том же смысле, что и поверхность планеты или звезды, например.

Но так то за всем этим очень много матана, и область эта ТЁМНАЯ, так что я более чем уверен что что-то упускаю.
Аноним 15/04/20 Срд 22:20:22 558542289
>>558531
Мне кажется для всех людей эта область тёмная. Хотя бы из за того что ближайшая ЧД в ебенях. Может какой-нибудь зашореный учёный сверх высокого класса и угадал какие-то параметры но без подтверждений так сказать в живую, эта угадайка малозначима.
Аноним 15/04/20 Срд 23:56:37 558550290
Какие есть идеи как сделать долговременный венероход, по поверхности?
Аноним 16/04/20 Чтв 01:08:26 558561291
Arch2O-Sttandbe[...].jpg 1667Кб, 1800x1200
1800x1200
Аноним 16/04/20 Чтв 01:31:39 558562292
>>558561
Эти хуйни чем-нибудь лучше сферического перекати-поля в пустыне? Они хотя бы переменным галсом идти могут?
Аноним 16/04/20 Чтв 07:41:51 558570293
>>558506
>линзирование
Это.

В что по поводу чёрных дыр? Свет их огибает так же и идёт дальше или это иллюзия?
Аноним 16/04/20 Чтв 08:29:32 558571294
>>558561
была где-то новость что разрабатывают венероход без электронной начинки
вопрос, нафига, как фотки-видео таким делать
Аноним 16/04/20 Чтв 09:59:07 558576295
>>558531
>Строго говоря ЧД это не совсем объект, а область пространства вокруг объекта содержащего всю её массу - сингулярности. И "край", про который ты говоришь - горизонт событий - это не физический объект в том же смысле, что и поверхность планеты или звезды
Золотые слова, анон. Читаю тут про какие то жидкие чд и люди похоже действительно думают, что чд на планету похожа что ли, хз как они ее представляют. Все просто на самом деле ахуенно сложно есть точка в пространстве, чем больше ее масса. тем шире горизонт событий.
Аноним 16/04/20 Чтв 12:26:40 558584296
>>557673 (OP)
Я снова тут.

Солнце — холодное тело с горячей фотосферой. Механизм гравитации

Кто-нибудь читал? Прост если эта догадка верна, то причина невидимости чего-то после сброса внешней оболочки другая, плюс становится ясно, что Солнца не умирают, просто циклически выбрасывают газ вот таким вот образом.

Солнца вечные кароч
Аноним 16/04/20 Чтв 12:39:26 558585297
>>558584
Ну это, конечно, интересно, но почему по-твоему в центре Солнца не должны происходить термоядерные реакции? Даже если по-твоему ядро состоит в основном из железа, оно, скорее всего, всё равно будет вступать в термоядерную реакцию.
Аноним 16/04/20 Чтв 12:51:27 558589298
>>558585
Термоядерная реакция это когда БАХ И ВСЁ СГОРЕЛО, а не когда что-то медленно протекает, плюс вещества в Солнцах недостаточно не то что там на 1000 лет, а даже на год.

Вобщем я считаю, что термоядерная реакция не может протекать долго даже при таких казалось бы больших размерах Солнц.

Меня другое заинтересовало, в этой книге у солнца есть твёрдая оболочка, которая холоднее поверхности, и есть ядро, которое очень холодное. Так вот если сбросить два верхних слоя, то ядро будет почти чёрным.

Я чот всё больше убеждаюсь, что вокруг нас не мгновения, а вечные циклы, прям как в программировании.

Да, я в курсе, что это теория, но я так же чот уверен, что и у Юпитера есть твёрдая поверхность и атмосфера с каким-то не сильно большим давлением, как на Венере или даже Земле. Я так же в курсе про спектры, которые говорят о другом.
Аноним 16/04/20 Чтв 13:17:16 558593299
>>558589
Почему ты считаешь, что все термоядерные реакции в Солнце должны проходить мгновенно, я так и не понял.

Я сейчас просмотрел эту книгу, я блять так и не понял почему он считает, что там должен быть какой-то холодильник, он вообще в курсе, что энергия может передаваться излучением?
Если Солнце способно создавать такое магнитное поле, то почему там нет термоядерных реакций? И почему тогда ядро холодное, так ещё и под таким давлением? Почему он решил что зведы не умирают? как бы у нас есть пруфы, что звёзды умирают - сверхновые. Блять, слишком много хуйни.
В общем не читай книги всяких ебанутых шизиков.

Да, у Юпитера есть "поверхность" из металического водорода скорее всего, но она находится не сразу под атмосферой, а под океаном из того же самого водорода.
Аноним 16/04/20 Чтв 13:23:56 558598300
>>558585
>Даже если по-твоему ядро состоит в основном из железа, оно, скорее всего, всё равно будет вступать в термоядерную реакцию.
Охуенные истории. Железо не имеет критерия Лоусона и вступить в реакцию может только с поглощением энергии.
>>558589
>Термоядерная реакция это когда БАХ И ВСЁ СГОРЕЛО, а не когда что-то медленно протекает, плюс вещества в Солнцах недостаточно не то что там на 1000 лет, а даже на год.
Охуенные истории. Пруфы будут?
>>558531
> у ЧД нет теоретического предела массы/размера
Вообще есть теории насчёт кварковых звёзд.
Аноним 16/04/20 Чтв 13:25:53 558599301
>>558593
Потому что только так они и протекают и довольно быстро весь водород перейдёт в гелий.

Касаемо магнитного поля, то у автора посыл простой - оно не может быть у настолько горячего ядра.

Мне нравится его теория, она очевидно не на 100% верна, но очень интересна. До этого натыкался на теорию про громадный кусок урана в ядре солнца, который все эти чудеса и творит.

И да, тебя не смущает разве тот факт, что поверхность Солнца это всего лишь 6000 Кельвинов? Если бы в нутри была бы горелка, то и на поверхности тоже, ведь излучение же, о котором ты говоришь не способно так быстро охладить такое тело.

>>558598
>Охуенные истории. Пруфы будут?

Да - царь бомба, у которой температура вообще 100 000 000 Градусов Цельсия. Помнится были опасения, что вода в океане продолжит дело сей бомбы и планета превратится в груду обломков.
Аноним 16/04/20 Чтв 13:28:26 558601302
>>558599
>Помнится были опасения
У кого, у пациентов Кащенко?
Аноним 16/04/20 Чтв 13:29:33 558603303
>>558599
> ведь излучение же, о котором ты говоришь не способно так быстро охладить такое тело.
Формулы, Билли, расчёты, а не просто "яскозал не может".
Аноним 16/04/20 Чтв 13:30:42 558604304
Аноним 16/04/20 Чтв 13:31:38 558606305
>>558603
Олсо
>Мощность, выделяемая различными зонами ядра, зависит от их расстояния до центра Солнца. В самом центре она достигает, согласно теоретическим оценкам, 276,5 Вт/м³. Таким образом, на объём человека (0,05 м³) приходится выделение тепла 285 Ккал/день (1192 кДж/день), что на порядок меньше удельного тепловыделения живого бодрствующего человека. Удельное же тепловыделение всего объёма Солнца ещё на два порядка меньше.
Педивикия такой себе пруф, но как указание немогликам сгодится.
Аноним 16/04/20 Чтв 13:32:59 558609306
>>558599
>Помнится были опасения, что вода в океане продолжит дело сей бомбы
Ага, а если обмотать Землю урановой проволокой, случится ядерный взрыв. Давление в >1000000 атмосфер откуда в океане возьмётся?
Аноним 16/04/20 Чтв 13:33:22 558610307
Аноним 16/04/20 Чтв 13:34:02 558611308
>>558422
0. Это же тупых вопросов тред?
1. Из чего изготовлена эта перегородка, в которой щели проковыряны? Уж не из ядер ли какого-нибудь металла, окружённых электронным облаком, которое влияет на угол движения мимопролетающих корпускул?
2. Не излучают ли во время опыта сами эти щели? Электрон ведь может и поглотиться ненароком, а то и переизлучиться.
Аноним 16/04/20 Чтв 13:37:20 558613309
изображение.png 84Кб, 1584x466
1584x466
Аноним 16/04/20 Чтв 13:38:38 558615310
14959096335150.jpg 3619Кб, 3200x4000
3200x4000
>>558571
>без электронной начинки
Аноним 16/04/20 Чтв 13:53:50 558618311
>>558613
Я, каюсь, не очень хорошо знаю физику звёзд, но мне кажется, что нельзя рассматривать излучение ядра просто через Стефана-Больцмана, учитывая тот факт, что большая часть энергии Солнца уходит на подержание термояда и проёбывается в зоне лучистого переноса.
Олсо, тогда Земля вообще не должна существовать по такой логике, так как мы находимся внутри солнечной короны с температурой в миллионы К.
Откуда взялось Eя?
Аноним 16/04/20 Чтв 13:56:04 558619312
изображение.png 12Кб, 1298x74
1298x74
Аноним 16/04/20 Чтв 14:10:43 558624313
1587035443060.jpg 277Кб, 700x868
700x868
>>558508
Обобщим вопрос: а можно ли установить изнутри черной дыры, что ты находишься в ЧД?
Аноним 16/04/20 Чтв 14:30:55 558631314
>>558531
>и область эта ТЁМНАЯ
Ясен хуй темная. Все почему-то забывают о парадоксе рождения вселенной.
В те лихие времена явно происходило некоторое дерьмо, что идёт совершенно вразрез с тем, что мы знаем о черных дырах.

Начнем с того, что большой взрыв вообще не должен был произойти.
Потому что вся масса вселенной была сосредоточена далеко за радиусом Шварцшильда.
Так какого хуя оно ебануло-то блять?
Да ещё и так, что образовалось пространство-время и вообще творилась какая-то дичь.

Так вот, это я к чему.
А с чего все взяли, что те законы, которые к этому привели, куда-то исчезли?
А если они никуда не исчезали, значит мы ещё дохуя чего не знаем, что происходит с веществом, энергией и пространством-временем в сверхкритических состояниях.
Аноним 16/04/20 Чтв 14:31:22 558632315
aX5RcWE6A.jpg 39Кб, 720x720
720x720
сап космач, я вас категорически приветствую
тут произошёл спор в конфе одной на космические темы, где мне кинули вот это видео https://www.youtube.com/watch?v=7CPlOe5ZAec доказывая, что у нас неправильная солнечная система, как минимум странная и нетипичная, как максимум искуственно созданная или подрихтованная. видос посмотрел, по ржал с параноика, но вот с 10:54 говорится что по формуле всемирного тяготения луну должно было бы давно оторвать, и я из-за незнания формул ничего противопоставит не могу. как поставить умника на место? реквестирую формулы с рассчётами
Аноним 16/04/20 Чтв 14:33:19 558634316
>>558624
Нет. Только один признак тому может сказать, что да - ты не сможешь достигнуть границы ЧД и вылететь за её пределы. Но ты из границы вселенной не можешь вылететь, так что даже если внутри ЧД, сейчас это абсолютно недоказуемо.
Аноним 16/04/20 Чтв 14:37:51 558635317
>>558632
>крамола
Вот можешь просто гринтекст ему отправить. Если он серьёзно относится к этому филиалу рен-тв в его худшие времена на ютубе, то никакие аргументы не помогут.
Аноним 16/04/20 Чтв 14:40:40 558638318
Аноним 16/04/20 Чтв 14:54:55 558640319
>>558632
>но вот с 10:54 говорится что по формуле всемирного тяготения луну должно было бы давно оторвать
Ебанько на твоем видео надо отправить в школу обратно, разбираться почему спутники не падают с орбиты на Землю, хотя та их притягивает. А Земля не падает на Солнце. потому что они достаточно быстро движутся вбок, чтобы не успевать упасть, это и есть орбитальное движение
Аноним 16/04/20 Чтв 15:03:37 558642320
>>558360
Аккуратно игнорируем адекватные ответы. Вся суть зогачебыдла
Аноним 16/04/20 Чтв 15:10:01 558643321
изображение.png 67Кб, 1920x1446
1920x1446
>>558632
>видос посмотрел, по ржал с параноика
>и я из-за незнания формул ничего противопоставит не могу

А может лучше сделать то, что порекомендовал автор видео?
Аноним 16/04/20 Чтв 15:10:50 558644322
>>558640
>потому что они достаточно быстро движутся вбок, чтобы не успевать упасть, это и есть орбитальное движение

Тащи расчёты.

Подсказка, сила центробежная.
Аноним 16/04/20 Чтв 15:20:08 558646323
ys1.jpeg 158Кб, 800x591
800x591
>>558644
>сила центробежная
Аноним 16/04/20 Чтв 15:21:49 558647324
>>558362
Аккуратно игнорируем адекватные ответы. Вся суть зогачебыдла x2
Аноним 16/04/20 Чтв 15:25:51 558648325
>>558412
>>558426
Не надо пытаться такую хуйню пытаться продать как интуитивную, блядь. Никто не купит кроме шизиков.
Аноним 16/04/20 Чтв 15:26:25 558649326
>>558646
Мне понятен твой шок.

Но вот тебе вопрос из практики.

Ты когда-нибудь пытался сделать уравновешенную систему на притяжении постоянных магнитов?
Аноним 16/04/20 Чтв 15:27:19 558650327
>>558648
Я по дискавери это лично видел, там фотон был показан летящим кусочком плоской синусоиды.
Аноним 16/04/20 Чтв 15:27:26 558651328
>>558643
Ну и что ты тут высрал? Как этот расчёт поддерживает ту шизню из видео? Хочешь смысла - просто добавь воды центробежную силу по школьной формуле.
Аноним 16/04/20 Чтв 15:28:31 558652329
>>558398
Неправильно понял. Что ты засунешь за горизонт событий - никогда больше не вернется, оно отсечено навсегда из этого мира.
Аноним 16/04/20 Чтв 15:28:53 558653330
>>558644
Дебил, физика в школе не началась ещё?
Аноним 16/04/20 Чтв 15:31:16 558655331
image.png 400Кб, 1600x900
1600x900
Аноним 16/04/20 Чтв 15:33:33 558656332
>>558651
>>558653

И как она должна помочь? Тебя не смущает почти двухкратная разница? Ты в курсе, что твоё утекание в бок закончится всё равно притяжением к Солнцу? Более того, центробежная сила сама по себе имеет вектор по касательной к траектории, что только ускорит улёт Луны с орбиты Земли.

И да, не надо плес тут херни с батутом и шариками.
Аноним 16/04/20 Чтв 15:35:34 558657333
>>558656
Что ты вообще несёшь? Планета движется вбок, гравитация её тянет перпендикулярно вектору скорости, создаётся центростремительное ускорение, планета движется по окружности.
Аноним 16/04/20 Чтв 15:39:05 558658334
>>558657
И что? Посмотри расчёты ещё раз! Разница в притяжении двухкратная и Солнце тоже ускорение в бок даёт Луне.

В любом случае с такой разницей Солнце должно перетянуть Луну.

Касаемо устойчивых систем, тут явно никто не пытался, вся суть теоретиков.

Так вот - на притяжении невозможно создать устойчивую систему, всегда будет перекос в одну из сторон. Это значит, что идея существования Солнечной системы на притяжении глупа в в принципе. Ещё раз, это невозможно, совсем.
Аноним 16/04/20 Чтв 15:56:47 558663335
>>558658
Какие расчёты? Я не собираюсь смотреть на расчёты дебила, который не осилил программу за 9-й класс.

>Солнце тоже ускорение в бок даёт Луне.
Солнце даёт Луне ускорение в сторону Солнца, а не вбок. Да, оно больше, чем ускорение, сообщаемое её Землёй, на основании этого, например, некоторые учёные в своё время предлагали считать Землю-Солнце двойной планетой солнечной системы, но это хуёвая идея, плохое определение двойной системы. Никуда Солнце Луну не должно перетянуть, Луна движется вместе с Землёй по орбите.

>Так вот - на притяжении невозможно создать устойчивую систему, всегда будет перекос в одну из сторон. Это значит, что идея существования Солнечной системы на притяжении глупа в в принципе. Ещё раз, это невозможно, совсем.
Невозможно вдолбить в твою тупую голову какие-либо знания, а устойчивые системы возможны, этим вопросом занимались дохуя кто, ещё с 18 века, сейчас у нас симуляции есть поведения планетных систем, в том числе и нашей. Она, кончено, хаотичная на больших масштабах, но ничего сверхестественного в ней нет. "Перекос" блядь, ты даже нормально не понимаешь, как работают орбиты, а что-то пиздишь.
Аноним 16/04/20 Чтв 16:06:08 558668336
>>558651
Какую ещё, в пизду, центробежную силу по школьной формуле? У тебя, блджад, небесные тела в космическом пространстве, они взаимодействуют только посредством сил гравитационного притяжения.
>>558658
>Солнце тоже ускорение в бок даёт Луне
Ну ты тугой. Для шаров, коими вполне можно считать звёзды и планеты, сила притяжения направлена к центру притягивающего тела.
>Солнце должно перетянуть Луну
Солнце притягивает и Змел. Забыл штоле? И Земля движется по орбите вокруг Солнца.
>идея существования Солнечной системы на притяжении глупа
А хотя а, ты шизик же.
Аноним 16/04/20 Чтв 16:11:40 558670337
>>558663
>а устойчивые системы возможны, этим вопросом занимались дохуя кто, ещё с 18 века

Неси тогда пример магнитной потенциальной ямы, рабочий только, а не на бумаге.

>Я не собираюсь смотреть на расчёты дебила, который не осилил программу за 9-й класс.

А конечно смех, там всё по формулам.

>>558668
Беда в том, профессор, что что-то задало и поддерживает орбитальные скорости планет, объяснять в духе ну это гравитация кароч, не уместно, выше тебе реальность, сила больше у Солнца, если этот процесс даже с угловой скоростью в масштабе сделать на магнитах всё схлопнется сразу же.

Я понимаю что вы с точки зрения теории пишете и у вас там всё просто замечательно, но я пробовал на притяжении магнитов создать устойчивую систему, в теории - да, на практике это невозможно.
Аноним 16/04/20 Чтв 16:15:13 558671338
>>558656
>центробежная сила сама по себе имеет вектор по касательной к траектории
Нет. Она направлена противоположно центростремительной. На самом деле центробежная сила фиктивная, но это слишком сложная тема для ответа с мобилки.
Но для расчета посчитай силу притяжения Луны Солнцем и посчитай, подставив даже не заморачиваясь с вращением орбитальную скорость земли, центробежную для Луны по школьной формуле эм вэ квадрат делить на Р, и у тебя все сойдётся.
Аноним 16/04/20 Чтв 16:19:02 558672339
>>558668
>они взаимодействуют только посредством сил гравитационного притяжения.
Чей Бог отменил в космосе силы инерции? Пойдём разберёмся, отвесим пиздюлей за такую хуйню.
Аноним 16/04/20 Чтв 16:19:14 558673340
>>558671
Проблема не в центробежной силе, Луна не разогналась и летит, а в том, что что-то явно поддерживает устойчивую систему так, что её нельзя нарушить, вот о чём я.

То есть орбита и скорость по ней это лишь следствие, но не убегания и притяжения, а чего-то ещё.

Я даже вангую, что для системы Земля-Луна, притяжение Солнца вообще не имеет значения, точнее оно имеет значение только на Землю.
Аноним 16/04/20 Чтв 16:20:37 558674341
>>558670
>Неси тогда пример магнитной потенциальной ямы
А чо не сильного или слабого взаимодействия? Если мешать всё в кучу, так уж всё сразу.
Аноним 16/04/20 Чтв 16:21:07 558676342
>>558632
Посмотрел эту хуитку, давай по полочкам всё тебе разложу.
>другие системы не похожи на нашу
Здесь во-первых есть detection bias: системы, похожие на нашу, гораздо труднее обнаруживать. Во-вторых, да, возможно, такое расположение планет, как у нас, является относительной редкостью. В этом нет ничего странного, если предположить, что именно такое расположение повлияло на вероятность нашего появления.
>формулка Тициуса-Боде
Эту хуету обнаружили ещё где-то в 18 веке, она давно опровергнута. Это совпадение, которое кажется более точным и невероятным, чем оно на самом деле является.
>симуляция
Да, может быть мы в симуляции. А может нет. А может те, кто создали симуляцию, сами в симуляции. Хуй знает, это всё вилами по воде писано, как с богом примерно. Единственное, о чём реально можно рассуждать, это об одном конкретном типе симуляции - симуляции своей истории. То есть если наши потомки симулируют нас. В этом (и только в этом) контексте верно следующее рассуждение: или а) такая симуляция окажется невозможной; или b) ненужной; или c) мы скорее всего живём в такой симуляции. При этом если так, то, к примеру, наша наука всё равно даёт верные ответы на вопросы о мире, ведь симулируется мир, такой же, как мир авторов симуляции.
>тёмная материя и тёмная энергия
Это вообще разные вещи.
>цитата Язева
Это просто вообще пиздец, более вопиющего вырезания из контекста я в жизни не видел. Просто скажу, что Язев не дебил, и ни в какое искусственное происхождение солнечной системы не верит.
>конспирология про Марс
Просто нахуй, даже пояснять нечего.
>про Луну
Хуета, уже пояснили в треде.

>>558670
>что что-то задало и поддерживает орбитальные скорости планет
Идиот блядь. Задало формирование солнечной системы, иди читай, как это происходило. А поддерживает инерция нахуй.
>Неси тогда пример магнитной потенциальной ямы, рабочий только, а не на бумаге.
Что ты блядь вообще имеешь в виду? Магнитное поле не является потенциальным, дурень.
Аноним 16/04/20 Чтв 16:22:08 558677343
>>558672
>Чей Бог отменил в космосе силы инерции?
Бог инерциальных систем отсчёта Нь-тон.
Аноним 16/04/20 Чтв 16:24:07 558678344
>>558676
>Задало формирование солнечной системы, иди читай, как это происходило.

Ты серьёзно сказания на тему выдаёшь за истину что ли?

>>558674
Для начала магнитного.
Аноним 16/04/20 Чтв 16:24:39 558679345
>>558670
>что что-то задало и поддерживает орбитальные скорости планет
Рандом задал, а удерживает инерция.
Аноним 16/04/20 Чтв 16:29:21 558680346
>>558678
>Ты серьёзно сказания на тему выдаёшь за истину что ли?
Да, я же не дебил. А вот тебе следовало бы проследовать отсюда нахуй. Этот тред для тех, кто хочет получить ответы на вопросы, а не спорить с азбучными истинами.
Аноним 16/04/20 Чтв 16:32:37 558681347
>>558680
>а не спорить с азбучными истинами

Фотографии развития планеты с нуля сюда неси, азубчные истины, ну-ну.
Аноним 16/04/20 Чтв 16:36:18 558683348
>>558681
Гугли protoplanetary disks photos.
Аноним 16/04/20 Чтв 16:41:43 558684349
>>558670
>что-то задало и поддерживает орбитальные скорости планет
Конечно задало. Изначально на месте Солнечной системы было немного неоднородное облако молекулярного газа. Оно стянуло к себе окружающий газ, остыло и схлопнулось под действием собственной гравитации, образовав Солнце. Но из-за изначальной неоднородности облака вещество тянуло в немного разных направлениях (с небольшой боковой скоростью относительно барицентра), поэтому при приближении к будущему Солнцу потенциальная энергия вещества стала размениваться на кинетическую, боковая скорость увеличивалась. (чем ближе к центру, тем быстрее крутишься). Образовался опять же немного неоднородный вращающийся протопланетный диск, в котором точно таким же образом сформировались протопланеты. Которые потом долго сталкивались друг с другом, слеплялись и в конце концов выжило лишь то, что мы видим сейчас - планеты, луны, астероиды и прочие тела, крутящиеся вместе с Солнцем вокруг общего барицентра и вокруг друг друга по сложным траекториям.

>всё схлопнется сразу же
>я пробовал на притяжении магнитов создать устойчивую систему
>на практике это невозможно
Анон, это тред тупых вопросов. Если ты хочешь задать тупой вопрос и понять на качественном уровне как это работает - пожалуйста. Если ты хочешь найти трибуну, попиздеть на ней про заговоры и цивилизации и кого-то переубеждать на уровне /б/ - проследуй нахуй.
Аноним 16/04/20 Чтв 16:54:57 558686350
>>558677
А почему тогда третья инерциальная сила называется ньютоновской? Ему не сказали наверное, сволочи.
Аноним 16/04/20 Чтв 17:01:46 558688351
>>558686
Хуй знает, кем она там называется, инерциальными силами называют силы, возникающие в неинерциальных системах отсчёта.
Аноним 16/04/20 Чтв 17:11:24 558689352
Спейсач, во что ты превратился? Нахуя вы спорите с долбоёбом, для которого КРАМОЛА БЛЯТЬ является нормальным источником информации?
Аноним 16/04/20 Чтв 17:22:01 558694353
>>558689
>Мы не проверяем факты
>Мы диктуем понимание мира
>За экспериментами в зог проследуйте

Эй мудрец! Когда гравицапу ждать? Напоминаю, ОТО скоро соточка лет будет.
Аноним 16/04/20 Чтв 17:27:18 558695354
>>558694
Какая связь между гравицапой и ОТО, еблан?
Аноним 16/04/20 Чтв 17:28:38 558696355
ЗАКРЫВАЙТЕ РАЗДЕЛ НА КАРАНТИН.
В НЁМ РАК.
Аноним 16/04/20 Чтв 17:28:58 558697356
>>558694
Лучше БэТэГхэ жди. На той же крамоле уже кучу прототипов показали.
Аноним 16/04/20 Чтв 17:57:35 558700357
>>558694
>Это тред тупых вопросов?
>А правда, что вся физика начиная с ньютона - пиздеж полный? Вообще вся!
>Что вы пиздите, ничего работать не будет, невозможно, я тут магниты покрутил! Вся физика - полный пиздежь!
>Что значит "пиздуй в зогач?"
Аноним 16/04/20 Чтв 18:12:50 558701358
Аноним 16/04/20 Чтв 18:47:35 558712359
>>558649
Во-первых, магниты имеют два полюса и их поле не сферически симметрично, это не аналог гравитации. Магнитные монополи принесешь — сделаю.

Электрические поля вот почти полный аналог, там на изичах всё делается, будут и орбиты, и спутники, и законы Кеплера.
Аноним 16/04/20 Чтв 18:49:50 558713360
>>558701
Если ты говоришь об электромоторе, то электромотор существует. Какое практическое применение в этом вращении здоровенных устройств,я не представляю. Катушки наматывать?
Аноним 16/04/20 Чтв 18:52:27 558714361
Аноним 16/04/20 Чтв 18:52:48 558715362
Что произойдет, если пиздануть солнечного лошка юпитерианским камнем? Сильно желтый отмеган оподливится во все стороны излучениеи?
Аноним 16/04/20 Чтв 19:14:25 558721363
>>558712
>Магнитные монополи принесешь — сделаю

Не существуют

>там на изичах всё делается

Мне реально работающие примеры машин тащи, а не сказки

>>558713
Это генератор. Почему ты проигнорировал, что там вращение безконтактно снимается, так же как и сами валы принуждаются к вращению также безконтактно?

И обращаю внимание, это реально работающий агрегат.
Аноним 16/04/20 Чтв 19:17:02 558722364
Homeworld.png 2513Кб, 2078x1536
2078x1536
>>558715
Произойдёт пуканьеро эль бомбардиньо с полыханием и фонищей такой, что совеременная венерианская атмосфера от поверхности будет отслаиваться, а ещё - если ты имеешь в виду просто ни с того ни с его по Солнцу ёбнет Юпитер, без учёта очевидно вредоносных для структуры СС перемещений Юпитера, тогда ещ орбиты будут дрожать как гитарные струны во время гитарного соло некоторое время, пока добавочная стоимость по Марксу масса не распределится равномерно по имеющимся масс-объёму Солнца и не устаканится
И вряд ли это как-то существенно повлияет на Солцне - ни светимость, ни объём, ни масс (ну почти), ни спектр излучени, класс звезды не изменятся просто солнышко будет едва отличимо больше, сперва побугуртив коронарным вспышками и радиацией во все стороны
Аноним 16/04/20 Чтв 19:27:36 558723365
Cyclotronmotion[...].jpg 1471Кб, 2939x1890
2939x1890
>>558721
>Не существуют
Какие ваши доказательства?

>Мне реально работающие примеры машин тащи

Любой аппарат, где электроны электромагнитным полем отклоняются.
Аноним 16/04/20 Чтв 21:42:02 558748366
Так блэд, посоветуйте книг про то как осваивался космос. Если есть такая где паралельно пишут про Америку и СССР то вообще супер
Аноним 16/04/20 Чтв 21:46:51 558749367
Аноним 16/04/20 Чтв 22:01:24 558750368
>>558721
Ну да. Если вращать электромотор он будет производить электричество. Полярность только поменять нужно. Но на видео было показано, как электроэнергия преобразуется в механическую, а не наоборот. И да это делается бесконтактно посредством магнитных полей. Вон там даже написано что там 480 вольт подключено к электромотору.
Аноним 16/04/20 Чтв 22:58:58 558758369
>>558748
Очевидный Черток Ракеты и Люди крайне рекомендуется для понимания истоков и той и другой программы, и тем более дневники Каманина. Обязательно Flight: My Life in Mission Control Криса Крафта и Failure is not an Option Джина Кранца. Ещё есть The Red Rockets' Glare: Spaceflight and the Soviet Imagination Азифа Сиддики, не участник, но очень достоверный историк. Эквивалента Чертоку по американской программе не знаю, особенно про ранние эксперименты с V-2, про Симона Рамо и про конкуренцию Титанов и Сатурнов сам бы читнул чего-нибудь.
Аноним 16/04/20 Чтв 23:00:43 558759370
>>558721
>безконтактно
Про бесколлекторные моторы постоянного тока ты не слышал? А про синхронные/асинхронные?
Аноним 16/04/20 Чтв 23:12:15 558762371
>>558758
>Черток
95% люди, про ракеты дай бог 5%.
Аноним 16/04/20 Чтв 23:40:29 558764372
>>558762
Инженерных баек там до жопы, и точно не 5%. Собственно как ты себе иначе представляешь любые мемуары, как не про людей? Да и вообще любую историю. У него во многом ключевые сведения про освоение. Если тебе чисто устройство йоб нужно, это не книги надо читать, а инфу собирать по справочникам и инету.
Аноним 16/04/20 Чтв 23:48:39 558766373
>>558762
Удваиваю. Вся книга из перечислений кто куда кем был назначен, за что сидел и куда расстрелян. Пришлось из-за этого дропнуть на 2/3 прогресса, ибо конца этому не предвиделось. Честно говоря, не понял, почему на это всё так дрочат в /spc/, но конец Второй мировой и её влияние на отрасль в целом неплохо описаны, пусть даже технических подробностей минимум.
Аноним 17/04/20 Птн 11:17:18 558784374
Может ли вырасти что нибудь на Марсе в открытом грунте, если непрерывно поливать так, чтобы вода не успевала выкипать?
Плюс включить мощные лампы для подогрева грунта, и чтобы вода не намерзала ледяной коркой.
Аноним 17/04/20 Птн 11:19:36 558785375
>>558784
Нит, единственная хуйня которая не задохнется - анаэробные бактерии. Водички навези и накидай этих сук вариться в окиянах.
По хорошему надо накидать комет с куйпера, чтоб и воды занести и азотику, а то на одном со2 мало кто сможет жить.
Аноним 17/04/20 Птн 11:45:17 558787376
>>558785
С хуя там растения задохнуться-то должны? Почти 100% углекислый газ в атмосфере.
Аноним 17/04/20 Птн 11:47:52 558788377
>>558787
Давления нет, читай что в вакууме.
Ну растопишь дохуя СО2, каким-то хуем создашь 1атм чистого СО2 - растения задохнутся нахуй, на земле таких нет которые на чистом СО2 могут жить.
Аноним 17/04/20 Птн 13:01:28 558794378
>>558695
Ну, ОТО не дает гравицапу, а эфирщики дают, вот и думой.
Аноним 17/04/20 Птн 13:02:22 558795379
>>558788
Растения же дышат углекислым газом и срут кислородом.
Аноним 17/04/20 Птн 13:07:48 558797380
>>558795
Plants have aerobic metabolisms. In an atmosphere of pure CO2, they would suffocate. This would be mitigated somewhat by the relatively small amounts of oxygen they could produce by photosynthesis before their metabolisms shut down because their mitochondria couldn't metabolize sugars.
Аноним 17/04/20 Птн 13:19:02 558799381
Аноним 17/04/20 Птн 13:23:30 558800382
а правда что гагарину на луне кто то сказал фразы из корана?
Аноним 17/04/20 Птн 13:31:42 558802383
>>558799
Я же уже сказал.
У вас по биологии не проходили еще, что растениям нужен кислород?
Еще читанул, что от СО2 кислотность резко повысится, что тоже убивает краба.
Но самое главное, что надо понимать - отсутствие атмосферы. Земные растения не смогут дышать просто-напросто.
Аноним 17/04/20 Птн 13:34:21 558803384
>>558802
>растениям нужен кислород
Так они сами его выделяют, оло.
Аноним 17/04/20 Птн 13:35:40 558804385
>>558803
В банке ты можешь попробовать помидорки повыращивать, там может и успеют навыделять, а на целый марс они кислорода не напердят и задохнутся первой же ночью, ололоша.
Аноним 17/04/20 Птн 13:54:30 558807386
>>558803
Ты масштаб представь, маня. И физику газов тоже.

Вот ты рассадил огород свой на Марсе. Сколько кислорода эти растения выработают за 1 сол? За год? Хватит ли этого, что бы поднять уровень кислорода в атмосфере Марса с 0,01% до 22-23%?

Если реалистичные варианты рассматривать, без огорода на всю ёбаную планету - то кислород будет просто будет улетучиваться, практически бесследно - ведь О2 легче чем СО2, т.е. он будет улетать в верхние слои атмосферы, а твои растения будут сидеть как фуфелы в практически чистом углекислом газе, не видя даже собственноручно вырабатонного О2. Прежде чем сдохнут от его нехватки.
Аноним 17/04/20 Птн 13:57:50 558808387
>>558807
Там и самого СО2 недостаточно для растений, почти вакуум же.
Надо сперва газиков надуть, водички нассать, чтоб парниковый эффект погуще.
Без бомбежки кучей комет хуй такого добьешься, останется только сидеть в бочках как фуфелы. Может проще солетту у Венеры надуть и подождать пока она остынет?
Аноним 17/04/20 Птн 14:48:26 558813388
>>558808
Надо, кто ж спорит.

Ну или КУПОЛ построить. Немного попроще чем полноценный терраформинг. И быстрее.
Аноним 17/04/20 Птн 14:50:04 558814389
>>558813
Не будет ли купол мешать наводить марафет кометами?
Аноним 17/04/20 Птн 15:17:45 558816390
>>558808
А есть хоть намёки на то, что на Марсе есть нефть?
Если б была, то можно её сжигать. Кислород для сжигания добывать из воды, содержащейся в этой же нефти.
Всяко реалистичнее бомбардировок кометами и говномастероидами.
Аноним 17/04/20 Птн 15:35:47 558818391
>>558816
>А есть хоть намёки на то, что на Марсе есть нефть?
Намёков нет.
>Если б была, то можно её сжигать.
Для СО2 карбонатные минералы подходят, они по идее есть на Марсе. Но даже если вся поверхность из них состояла бы - надо было бы 10 метров пород по всей планете прокаливать.
Другой вариант - бурить мохол в надежде на то, что пародия на магму осталась и не придется сотни километров бурить, чтоб вулкан запилить.
Самая реалистичная схема по созданию атмосферы - это все же насрать кометами, их примерно порядка 10 000 надо. Заодно азот завезут, на месте его уже нет.
К тому моменту и махину для запила магнитного поля можно запиливать, с такими мощностями с которыми ты корабли-редиректоры комет построишь - это уже реально.
Аноним 17/04/20 Птн 15:53:49 558820392
>>558816
>Кислород для сжигания добывать из воды, содержащейся в этой же нефти.
Что-то уровня толкать машину руками до супермаркета чтобы загрузить продукты в багажник и толкать назад опять руками. Не ну а хуле, вместительный же багажник!
Аноним 17/04/20 Птн 16:14:08 558823393
>>558818
>К тому моменту и махину для запила магнитного поля можно запиливать, с такими мощностями с которыми ты корабли-редиректоры комет построишь - это уже реально.

БТВ, довольно реалистичный и относительно реалистичный концепт - это эдакий "магнитный щит" перед планетой. На каком-то из англоязычных ресурсов про такую тему читал.

Грубо говоря это станция, которая висит в точке L1 системы планета-звезда. Термоядерный реактор, или солнечные панели, или ещё хуй знает что - создают мощный ток, который гоняют по катушкам, которые создают мощное магнитное поле на пути солнечной радиации к планете. И оно тип защищает планету от лучей. Типа примерно так же, как луна закрывает солнце во время затмения - но для солнечной радиации.

Ясен хуй это довольно грандиозно, и от галактической внесолнечной радиации не защитит. Но всё же проще чем обматывать всю планету медной проволокой, или втыкать в неё гигантские электроды, или ещё что.
Аноним 17/04/20 Птн 16:14:32 558824394
>>558823
>относительно реалистичный
относительно выполнимый, ай мин
Аноним 17/04/20 Птн 16:19:42 558825395
>>558823
Ну нихуя ты америку открываешь. Уже было в сурвайвин марс.
Аноним 17/04/20 Птн 16:20:48 558826396
Аноним 17/04/20 Птн 16:28:06 558827397
>>558825
Бля, я подумал - нихуя в симуляторе аутистичных покатушек на ровере контента, а я с Take On Mars спутал. А Surviving это парадоксодрочильня с 9 ебаными длц.
Аноним 17/04/20 Птн 17:30:48 558830398
>>558823
Вангую, этот щит будет мотылять солнечным ветром как говно в проруби, даже если его чуть ближе к Солнцу вынести. А ионные движки для коррекции мало подходят, и придётся к этому щиту возить топливо.
Аноним 17/04/20 Птн 19:11:13 558835399
>>558814
Построить купол дешевле и быстрее чем кометами закидывать, на первое время сойдёт.
Аноним 17/04/20 Птн 19:15:16 558836400
>>558835
Я не против купола.
Но не распидорасит ли когда ты решишь завести атмосферы кометами?
Аноним 17/04/20 Птн 20:04:42 558847401
>>558836
>>558835
Кометы не нужны. На Марсе столь малая гравитация, что атмосфера с такого терраформинга один хуй проебётся очень быстро.
Зато на Марсе можно построить не просто купол, а ГИПЕР-КУПОЛ. Размерами в сотни, в тысячи километров и высотой с земную тропосферу. Этому способствуют два фактора: во-первых, низкая гравитация (высотные конструкции и тонкие фермы строить проще), во-вторых, окружающий вакуум. Проще говоря, надо найти ту грань, при которой избыточное давление 100кПа внутри купола уравновешивается тяжестью конструкций, и тогда масштабировать его можно хоть до усрачки. Хватит и на леса, и на озёра, и на всё что угодно.

При этом планета, 50% площади которой покрыто гипер-куполами, а остальные 50% практически вакуумом - это просто шикардос с точки зрения промышленности (где нужен вакуум) и транспортной логистики. Не нужны никакие гиперлупы, поезда-маглевы можно так пускать с охуенными скоростями, причём благодаря низкой гравитации маглев сожрёт меньше энергии. А в космос можно прямо с поверхности электромагнитной катапультой выходить, практически без затрат топлива.
Аноним 17/04/20 Птн 20:10:58 558849402
>>558847
>атмосфера с такого терраформинга один хуй проебётся очень быстро.
Миллионы лет.
И то без магнитного поля.
Если ты за десятилетия можешь кометами накидать газиков - какая это проблема вообще?
>ГИПЕР-КУПОЛ
НЕ НУ ЖЕН
Ему достаточно одной дырки, и внезапно внутри все дышат вакуумом. Куча изолированных соединенных куполов безопасней.
А вот космический лифт на Марсе проще будет.
>А в космос можно прямо с поверхности электромагнитной катапультой выходить, практически без затрат топлива.
Только если катапульта дохуя высокая.
Та жалкая пародия на атмосферу все равно будет сжигать твою ПН, и нужен будет теплощит. Они, на минуточку, на марсе пригождаются и работают для аэробрейкинга.
Аноним 17/04/20 Птн 20:28:19 558856403
>>558847
>на Марсе можно построить не просто купол, а ГИПЕР-КУПОЛ
Как строить будешь?
Аноним 17/04/20 Птн 20:30:38 558858404
>>558856
Возьму пики точены срублю ху...
Стоп, это для другого.
Он наверное наноматериалы изобретет.
Аноним 17/04/20 Птн 20:31:54 558859405
>>558849
>Миллионы лет.
Не миллионы. Может несколько тысячелетий, но может и быстрее. С увеличением плотности это время ускоренно сокращается. Суть в том, что даже 1атм для Марса это много, а человеку даже при 0.5атм хуёво.

>Ему достаточно одной дырки, и внезапно внутри все дышат вакуумом.
От конструкции зависит. Есть всякие варианты многослойных самозатягивающихся конструкций, которые разъебать можно разве что макроскопическим метеоритным ударом. Алсо, из-за громадного объёма от одной дырки это "внезапно" будет длиться неделями и месяцами, за которые любую дырку сто раз уже залепят.

>Только если катапульта дохуя высокая.
Нахуя, лол?
>Та жалкая пародия на атмосферу все равно будет сжигать твою ПН, и нужен будет теплощит. Они, на минуточку, на марсе пригождаются и работают для аэробрейкинга.
Марсианский аэробрейкинг и земной - это две большие разницы. Плотности отличаются где-то в 200 раз, во столько же раз тормозящие эффекты. Чтобы затормозить об марсианскую атмосферу - надо долго и нудно нарезать в ней кучу витков. При единомоментном выводе на орбиту ПН атмосферы даже не заметит из-за крайне малого времени воздействия.

Атмосфера Марса - это как если построить на Земле катапульту на высоте 30+км. Тогда и с Земли можно было бы за гроши тысячи тонн запускать.
Аноним 17/04/20 Птн 20:44:22 558860406
image.png 135Кб, 1077x433
1077x433
>>558859
>Не миллионы.
Миллионы. 1-10 для СО2
Но при терраформинге ты будет предотвращать эрозию и защищать атмосферу магнитным полем.
Даже 1 миллион лет это дофига, пополнять раз в тысячу лет не сложно, если изначально бар запилить сумел.
>От конструкции зависит. Есть всякие варианты многослойных самозатягивающихся конструкций
Пока даже небольших не запилили.
Когда ты можешь в такие гигаконструкции - ты уже можешь в полноценный полет большой автономности. Можно и атмосферу запиливать, перекантовываясь в небольших куполах.
>При единомоментном выводе на орбиту ПН атмосферы даже не заметит из-за крайне малого времени воздействия.
Если свысока пулять - да. С поверхности заметит, один хуй защиту мазать надо будет.
>это как если построить на Земле катапульту на высоте 30+км.
Только гореть корабли на орбитальной скорости начинают уже на 130км. Да, у сникерса орбитальная помедленнее, но и стартуешь ты ниже.
Аноним 17/04/20 Птн 20:45:40 558861407
>>558856
Обводишь десяток рабочих рамкой и отдаешь им приказ.
Аноним 17/04/20 Птн 20:47:12 558862408
Аноним 17/04/20 Птн 21:02:35 558864409
>>558859
>Не миллионы. Может несколько тысячелетий, но может и быстрее. С увеличением плотности это время ускоренно сокращается. Суть в том, что даже 1атм для Марса это много, а человеку даже при 0.5атм хуёво.

Нет. Планеты - это не теплицы, bruh. Они, сука, БОЛЬШИЕ. За тысячи лет ничего на них не происходит в таких объёмах, используй матан,

Марс теряет около 100 грамм атмосферы в секунду сейчас, по данным MAVEN. Масса его атмосферы - 2,5 х 10^16 кг. Это оч. мало по сравнению с массой атмосферы Земли, но всё равно весьма нехило. Если темп угона атмосферы ускорить в миллиард раз (это 100 тысяч тонн в секунду, весьма серьёзно, но я полагаю в пределах допустимого) - на потерю атмосферы уйдёт чуть меньше миллиона лет. Терять её всю не обязательно, конечно, но даже на потерю 15-20% массы атмосферы уйдут сотни тысяч лет. И это при условии что мы не трогаем массу атмосферы планеты, а только сдуваем её огромным вентилятором, что абсурдно - терраформинг и нормальное для человека давления подразумевает увеличение массы атмосферы тоже.
Аноним 17/04/20 Птн 23:55:41 558903410
>>557673 (OP)
Вселенная бесконечна или нет?
Аноним 17/04/20 Птн 23:58:22 558904411
>>558903
Обозримая - нет. А так вообще - неизвестно.
Аноним 18/04/20 Суб 00:00:55 558905412
>>558904
Какая смерть нашей Вселенной наиболее реальная?
Аноним 18/04/20 Суб 00:03:02 558906413
>>558903
Бесконечна в том смысле, что у нее нет границ. Ты можешь сколько угодно двигаться в любом направлении, и никакого "края вселенной" не встретишь. Насчет объема - неизвестно.

Примерная аналогия вселенной с конечном объемом: поверхность Земли: у Земли нет края, двигаться можно сколько угодно. Но площадь конечна.

Примерная аналогия вселенной с бесконечным объемом: плоскость в евклидовом пространстве.
Аноним 18/04/20 Суб 00:06:05 558908414
Аноним 18/04/20 Суб 01:13:43 558920415
Можно ли квантовые эффекты в макромире наблюдать?
Очень понравилась реализация в The Outer Wilds (кто играл - не спойлерите, самостоятельно расследовать охуенно).
Аноним 18/04/20 Суб 03:10:59 558925416
>>558906
Твой пример с Землёй неверный. Потому что твоё "нет границы у поверхности Земли" будет более-менее правдой только если мы рассматриваем эту поверхность в двухмерном пространстве. Потому что поверхность и правда двухмерная, на небольших масштабах. Но на больших она уже искривляется, однако само понятие кривизны отсутствует в двухмерном мире. Вот и получается у нас как бы бесконечная поверхность.
Но в двухмерном пространстве нет понятия объёма. Ты не можешь говорить: "У Земли конечный объём, но бесконечная поверхность". Или рассуждай про одно, или про другое.
Теперь, если этот пример расширить до описания Вселенной (как ты сделал), то получится вот что: мы говорим про трёхмерное пространство, значит если у Вселенной конечный объём, то и граница у неё тоже есть. Значит твоя аналогия не работает ни на уровне Земли, ни на уровне Вселенной.
Аноним 18/04/20 Суб 04:07:22 558927417
Будет какой то толк, если бомбардировать планеты нашими самыми выносливыми организмами? Глядишь через пару тысяч они на планету кислород завезут. Понятно что не для дыхания человека, но выживать другим организмам там будет попроще. Самым очевидным вариантом будет титан, вторым уже марс или Европа ( спутник Юпитера)
Аноним 18/04/20 Суб 04:20:38 558928418
>>558927
>Марс, Титан
Да, я знаю про попытки найти там что то свое, но можно хоть 1 спутником пожертвовать?
Аноним 18/04/20 Суб 04:24:08 558929419
>>558927
Можно. Тем самым убив возможную жизнь которая не исключено есть во всяких необычных местах любых тел солнечной системы. Ну или как минимум сделает её неотличимой от земной. По этому по возможности стараются избегать микробного заражения иных тел.
Аноним 18/04/20 Суб 04:26:39 558930420
>>558928
Это бессмысленно. Жертвовать тем что мы ни разу не видели - инопланетную жизнь иной структуры. Которая может привести ко многим открытиям, в том числе прикладного характера. В пользу мутной хуйни без задачь, которая случится хуй пойми когда.
Аноним 18/04/20 Суб 05:27:28 558933421
>>558927
Нет. Ни на одном небесном теле в Солнечной системе, помимо Земли, нет жидкой воды на поверхности, а она требуется вообще всей известной жизни. Солнечный свет не нужен, кислород не нужен, даже давление не так сильно нужно, но без жидкой воды жизни нет, это обязательное условие.

Единственный вариант на данный момент это насрать хемотрофами в гипотетические подлёдные океаны и ждать, пока забурлит.
Аноним 18/04/20 Суб 06:06:06 558935422
>>558925
Аналогия работает в п-в Вселенной совершенно прекрасно.
В случае с положительной кривизной ты прилетишь в то же место откуда начал.
В случае с нулеовй и отрицательной эта аналогия не нужна, ты до границ не можешь долететь никак даже если они и есть в каком-то физическом смысле - они абсолютно недостижимы.
Аноним 18/04/20 Суб 06:07:27 558936423
>>558929
>По этому
>>558930
>задачь
Пиздец.

Только этот >>558933 нормально ответил.
Аноним 18/04/20 Суб 09:10:32 558945424
>>558936
Лол, два анона все кратко и по делу сказали, третий высрал хуйню с
>гипотетическими подлёдными океанами
вооо это подходит моему манямирку

Какие нахуй гипотетические океаны, Энцелад попукивает вполне негипотетическими гейзерами, там 100% жидкая вода есть.
Аноним 18/04/20 Суб 11:41:13 558952425
А почему нет треда Земли?
Аноним 18/04/20 Суб 11:42:06 558953426
>>558952
Все остальные доски про неё. На ней слишком дохуя всего происходит.
Аноним 18/04/20 Суб 11:51:01 558954427
>>558925
Начнем с того, что вселенная не трехмерна. Поэтому объем будет играть ту же роль, что и площадь в случае аналогии с Землей
Аноним 18/04/20 Суб 12:06:28 558957428
GatewaySpaceSta[...].jpg 2676Кб, 4000x2250
4000x2250
Объясните нахуй нужна орбитальная станция на орбите Луны?

Во-первых, в чем отличие от МКС? Только местоположением?
Это абсолютно то же самое говно с научной точки зрения.
Да, можно отработать все эти системы доставки, сборку нового поколения, новые корабли, только это все дрочево вприсядку ради дрочева вприсядку.

Еще слышал, мол, оттуда удобно летать до Марса.
Нет блять, тормозить и скруглять орбиту ради пристыковки к консервной банке - это нихуя не удобно, это пустая трата дельты.

Может быть, она как-то нужна для предстоящих миссий на поверхность?
Может быть, в случае каких-то неполадок на поверхности можно побыстроляну метнуться кабанчиком к станции и отсидеться там?
Или пристикованный к станции корабль сможет эвакуировать с лунной орбиты?
Так нет блять, нихуя из этого.
Ее орбита планируется настолько высокой и ебнутой, что чтобы устроить рандеву с рандомно взлетевшим куском говна с поверхности, может уйти недели 3.

Тем более, ничто не мешает иметь на этот случай на орбите Луны тупо тот же заправленный пустой запасной орион.

Короче какая-то максимально бесполезная дорогая хуйня.
Зато технически просто консервных банок нахуярить, имитация бурной деятельности.
Аноним 18/04/20 Суб 12:11:37 558959429
>>558957
А че рука всегда канадская? Муриканцы, европейцы не умеют? На шатал тоже канадцы делали?
Аноним 18/04/20 Суб 12:52:11 558963430
>>558957
>Это абсолютно то же самое говно с научной точки зрения.
Нет. И если ты не понимаешь почему с научной точки зрения орбиты 2 небесных тел являются одинаковыми, то с тобой не о чем разговаривать. Просто блядь мозгами подумай - давай представим что есть ученый Иванов, который изучает циклоны выглядывая из окна МКС, но внезапно МКС с Ивановым оказывается на орбите Луны - внезапно Иванов не может изучать циклоны , и вместо него было бы полезно иметь Петрова, который изучал бы кратеры на Луне. Две локации, две разных научных среды.
Аноним 18/04/20 Суб 12:52:38 558964431
>>558963
>с научной точки зрения орбиты 2 небесных тел НЕ являются одинаковыми
фикс
Аноним 18/04/20 Суб 13:09:38 558965432
u418130816.jpg 42Кб, 213x416
213x416
>>558953
И где про нее как про планету? Доски про всякую плесень.
Аноним 18/04/20 Суб 13:12:25 558966433
>>558965
В вопросе планетологии пиздуй в /sci/ спроси здесь, обсуждач по вопросу без треда тут вполне уместен.
Не так часто про неё спрашивают, вот треда про планету Грязь и нет.
Аноним 18/04/20 Суб 13:15:52 558968434
>>558963
Госпаде, твои кратеры прекрасно изучаются спутниками.

Вот база на поверхности — совсем другое дело. Там уже можно заниматься принципиально другими вещами, которыми ни спутник, ни ровер заниматься не может.
Аноним 18/04/20 Суб 13:17:13 558970435
>>558957
>Во-первых, в чем отличие от МКС? Только местоположением?
Тем что в отличие от МКС оно должно быть (по плану) гораздо более автономное и расчитанное на межпланетные перелеты. Нельзя просто взять МКС и пульнуть его на орбиту луны - оно там просто работать не будет. А DSG/LOP-G это марсианский/межпланетный корабль не отлетающий далеко от земли.

>Еще слышал, мол, оттуда удобно летать до Марса.
>Нет блять, тормозить и скруглять орбиту ради пристыковки к консервной банке - это нихуя не удобно, это пустая трата дельты.

Мог бы и дальше прикидыватся умным троллем но скруглять орбиту тебя выдало.

К марсу не получается летететь с LEO на ионниках, слишком долго придется жарится в поясах ван-аллена. Если лететь на водороде то мало того понадобится огромная сборка, так там водород выкипет, не успеем. Схема же с DSG/LOP-G позволяет выбираться из гравитационного колодца на водороде, используя стандартные третьи ступени. А затем спокойно лететь на ионниках.

Второй момент - LOP-G можно нормально снабжать коммерческими провайдерами, тяжелых ракет хватает для небольших грузовиков доставляющих эксперименты и мелкие запчасти. А это резко упрощает поддержание станции в рабочем состоянии, не нужно надеяться на единственный SLS. С такой же станцией на низкой орбите луны это не выйдет.

Третье DSG/LOP-G это компромисс между лунными и марсианскими лоббистами в наса. Которые до этого портили жизнь друг другу.
Аноним 18/04/20 Суб 13:20:09 558974436
>>558970
>но скруглять орбиту тебя выдало.
А это почему?
Аноним 18/04/20 Суб 13:20:17 558975437
>>558966
Куда постить и обсуждать новость что Земля была раньше плоской? https://2ch.hk/news/res/7344051.html
Или там остатки какого-то тела нашли в вулкане, интересно же.
>Не так часто про неё спрашивают
Это как-то глупо.
Аноним 18/04/20 Суб 13:21:14 558977438
>>558975
>Куда постить и обсуждать новость что Земля была раньше плоской?
В зогач, без вариантов.
>Или там остатки какого-то тела нашли в вулкане, интересно же.
Ну если что-то связанное с планетологией - то в тред новостей.
Аноним 18/04/20 Суб 13:22:27 558978439
>>558977
>В зогач, без вариантов.
Долбоеб?
Аноним 18/04/20 Суб 13:24:36 558981440
>>558978
Ты? Верящий в плоскую землю? Абсолютно безусловно. В зогач. А лучше в окно.
Аноним 18/04/20 Суб 13:25:13 558982441
>>558977
>в тред новостей
Схуяли?
Аноним 18/04/20 Суб 13:29:28 558987442
>>558970
>DSG/LOP-G это марсианский/межпланетный корабль не отлетающий далеко от земли
Хуя, сильное заявление.
У него хотя бы ядерный реактор будет?
Аноним 18/04/20 Суб 14:18:38 558994443
Аноним 18/04/20 Суб 15:19:12 559001444
>>558970
Эй, а ЧО ЭТО "не получится снабжать базу на низколунной орбите коммерческими носителями"? С каких таких хуёв? Тебе с низкой околоземной на низкую окололунную меньше 4 км/с надо. Давай, скажи мне сейчас, что на твою чудесную гало-орбиту нужно оче сильно меньше дельты! Ты не скажешь, ага, потому что это враньё.
Аноним 18/04/20 Суб 15:39:24 559004445
>>558994
А до Марса на гептиле лететь?
Аноним 18/04/20 Суб 15:42:31 559006446
гептильчик.png 416Кб, 999x635
999x635
>>559004
Алан Макс хочет волшебным образом метаниум сохранять на протяжении дохуя времени.
Я думаю, что это не невозможно.
Тем не менее, не вижу ничего сверхъестественного в теплом ароматненьком гептильчике, надолго хватит, обратно на нем можно, а пулять в сторону марса на еще не сопревшем водороде/метане.
Аноним 18/04/20 Суб 15:54:36 559010447
>>559006
А известна ли тебе такая аббревиатура как СПГ?
Аноним 18/04/20 Суб 15:59:43 559011448
>>559010
Солнцевская Преступная Группировка.
Спижженный Природный Газ.
А что?
Аноним 18/04/20 Суб 16:44:46 559024449
Как в ГфЯРД закрытого типа вообще возможно контролировать ядерную реакцию?
Аноним 18/04/20 Суб 16:52:07 559025450
>>559024
Заглушить возможно, я думаю, сделав его несколькоразовым, 2-3 маршевых хода, и это я ладонью в воздухе вожу. Скорей всего это одноразовая вещь, т.к. перезапускать может оказаться просто непрактично.
Что же до управления - тягой двигателя можно управлять.
Скоростью реакции - мне кажется, что внедрением поглотителей нейтронов в какой-то степени.
Я давно читал про это и материалов про такое видел немного, буду рад чего-то еще увидеть.
Аноним 18/04/20 Суб 20:58:57 559057451
>>558936
>Пиздец.
То ли ещё будет
Аноним 18/04/20 Суб 21:04:04 559059452
>>559057
Я пару раз пересматривал пост, т.к. мне казалось, что я не замечаю какую-то подъёбку в виде "толи", либо "будеть".
Иногда лучшая подъёбка - это её отсутствие.
Аноним 18/04/20 Суб 21:09:10 559061453
>>557858
>Что в Млечном пути существуют две разных звездных популяции
Это я и так вижу. Хули так-то? Какие объяснения предполагают для такого неравномерного распределения?
Аноним 18/04/20 Суб 21:12:48 559062454
>>557858
А почему магнезия в старых больше?
Аноним 18/04/20 Суб 21:31:39 559063455
>>559001
>Эй, а ЧО ЭТО "не получится снабжать базу на низколунной орбите коммерческими носителями"? С каких таких хуёв? Тебе с низкой околоземной на низкую окололунную меньше 4 км/с надо. Давай, скажи мне сейчас, что на твою чудесную гало-орбиту нужно оче сильно меньше дельты! Ты не скажешь, ага, потому что это враньё.

Не позорься дальше, а?

https://space.stackexchange.com/questions/23992/why-is-a-near-rectilinear-halo-orbit-proposed-for-lop-g-formerly-known-as-deep/24018

A study considering NASA SLS and Orion performances was carried out. Since SLS places Orion in a trans-lunar trajectory, Orion vehicle has to carried out orbital maneuvers to reach a orbit near the Moon, so the limiting factor will Orion's propellant load limitations. These makes difficult to access low-lunar orbits since an estimated budget of ΔVΔV=1250 m/s has to be enough for entering and leaving the orbit.

Analyzing the previous constraint, although for some epochs moon keplerian orbits were affordable, they were discarded because they were problemtic for continuous access. This reduces the candidates to orbits within the Restricted 3-Body Model: nominal L1/L2 halos (637-811 m/s), near-rectilinear halo orbit (NRO) (751-840 m/s) and distant retrograde orbit (DRO) (841-957 m/s).
Аноним 18/04/20 Суб 21:32:39 559064456
>>558974
>А это почему?

Потому что суть выбранной для LOP-G орбиты в том что, упрощая, это нескругленная орбита.
Аноним 18/04/20 Суб 21:33:50 559065457
>>559064
Точняк, я забыл про это, спасибо.
Аноним 18/04/20 Суб 21:34:45 559066458
>>558987
>Хуя, сильное заявление.
>У него хотя бы ядерный реактор будет?
>>559006
>Алан Макс хочет волшебным образом метаниум сохранять на протяжении дохуя времени.
>Я думаю, что это не невозможно.
>>559004
>А до Марса на гептиле лететь?
>>558994

"Дебилы блять"(C)

Лететь будет на SEP, solar-electric propulsion. Используя гептил(или что там у них) в тех случаях где ионники не справятся - запас хим топлива там есть и довольно большой.



Аноним 18/04/20 Суб 21:40:33 559067459
>>559062
>А почему магнезия в старых больше?
Лол. Во-первых, магния. Магний - продукт термоядерного "горения" в массивных звёздах на поздних этапах жизни.
Аноним 18/04/20 Суб 21:41:51 559068460
Аноним 18/04/20 Суб 21:47:16 559070461
>>559066
Че обзываешься, сука? Если бы все всё знали, тред бы не имел смысла.
Аноним 18/04/20 Суб 21:47:57 559071462
>>559068
В старых больше. СТрелка вверх.
Аноним 18/04/20 Суб 22:03:26 559077463
Аноним 18/04/20 Суб 22:04:00 559078464
Самоизолянты, можно помедленнеея записую. Вас дохуя в космос потянуло из четырёх стен? Корону не победили, но собрались пространство-время побеждать. Тред за неделю уебали, охуеть. Раньше такой хуйни не было.
Аноним 18/04/20 Суб 22:04:40 559080465
Аноним 18/04/20 Суб 22:04:41 559081466
Тут всё время пишут, что жидкий водород (и даже метан) быстро выкипают. Нихуя не пойму с 70-х годов разучились баки делать что ли?
Цитата из книги Джима Лоувелла и Джеффри Клюгера "Аполлон-13":
>Теплоизоляция баков была столь хороша, что если бы их наполнили обыкновенным льдом и оставили при комнатной температуре в 21 градус, то только через восемь с половиной лет лед бы растаял, и еще через четыре года вода нагрелась бы до комнатной температуры.
Аноним 18/04/20 Суб 22:06:54 559082467
>>559061
Легкие звезды живут долго, какие-нибудь красные карланы вообще живут дольше, чем Вселенная до сих пор. Поэтому такие звезды образовались и кукуют себе помаленьку. А всякие гиганты прожигают жизнь за миллиард лет, а то и десятки-сотни миллионов. Поэтому вот поколения звезд не совпадают. Тебя ж не удивляет, что поколение у людей - лет 20, а у дрозофил - неделя.
Аноним 18/04/20 Суб 22:09:02 559085468
>>559078
>Раньше такой хуйни не было.
Верни мочу, которая банит за любой пук, и будем загнивать как в старые добрые времена.
Аноним 18/04/20 Суб 22:10:10 559086469
>>559077
А этот пост >>559067
>Магний - продукт термоядерного "горения" в массивных звёздах на поздних этапах жизни.
Аноним 18/04/20 Суб 22:10:22 559087470
>>559081
Ну во-первых, ты не путай хуй с пальцем. Для жидкого водорода из-за его крайне низкой температуры тепловой поток через стенку будет в тыщщи раз сильнее.
Во-вторых, теплоизоляция, внезапно, весит. Причём нехило весит. Поэтому в ракетах, которые не должны летать в космосе по две недели, теплоизоляции НЕТ.
>>559085
Кажется, её перевели в wm
Аноним 18/04/20 Суб 22:12:08 559090471
>>559081
Физику школьную вспомни.
А если ты бак этот оставишь на орбите плутона, то лед вообще 2000 лет и более останется льдом, охуеть!

>>559085
За пук не было, за порашу справедливо, но медленно.
Аноним 18/04/20 Суб 22:23:26 559095472
>>559087
>Во-вторых, теплоизоляция, внезапно, весит
То есть на древние Аполлоны дохуя лишней массы влезало, а сейчас это, блядь, невозможно?
>>559090
>плутона
Плутон нахуй не нужен, надо до Марса и обратно на водороде, а тутошние васяны считают,что таких баков нет и быть не может, пиздец просто.
Аноним 18/04/20 Суб 22:24:15 559096473
>>559095
А сейчас это нахуй никому не нужно, дубинушка
Аноним 18/04/20 Суб 22:24:56 559097474
>>559095
>тутошние васяны считают,что таких баков нет и быть не может, пиздец просто.
Никто так не считает.
Водород будет дохуя быстро выкипать, блядь.
Сядь да сам посчитай, это несложно, седьмой класс формулы, достань справочник, должен быть недалеко.
Аноним 18/04/20 Суб 22:33:22 559102475
>>559095
Аполлоны же вроде на вонючке летали, или нет?
Аноним 19/04/20 Вск 00:53:41 559140476
aces1.jpg 61Кб, 941x709
941x709
>>559081
>Нихуя не пойму с 70-х годов разучились баки делать что ли?
Не, просто речь идёт о разных вещах. Сатурновские ступени были короткоживущими, их задача была пульнуть Аполлоны на отлётную траекторию, дольше нескольких часов они не жили все равно. S-IVB жила меньше 3 часов, а водород выкипал со скоростью 1100л/мин на Земле, и это при всей этой изоляции.

В концепте были проекты ступеней Сатурна, живущих до 10 дней, но они даже не дошли до какой-либо проработки.

>>Теплоизоляция баков была столь хороша, что если бы их наполнили обыкновенным льдом и оставили при комнатной температуре в 21 градус, то только через восемь с половиной лет лед бы растаял, и еще через четыре года вода нагрелась бы до комнатной температуры.
Дело не в теплоизоляции, никакая изоляция не поможет тебе продержать водород долго. Дело в тепловом балансе. Солнце жарит, движки жарят, фон поглощает тепло. При этом водород у тебя находится при температуре 21К, и обмен с фоном не очень-то эффективен. Для того, чтобы прожить больше нескольких часов, тебе нужно быстро сбрасывать тепло, нужны радиаторы и зонтик.

Поэтому например боинговский разгонник ACES для Вулкана довольно круто выглядит на бумаге - обещают 14 дней жизни с минимальным выкипанием водорода. Это реально дохуя для водорода. Достигается это у них двумя вещами: 1) использованием корпуса как радиатора и 2) Integrated Vehicle Fluids, они еще несколько лет назад её испытали вроде. Это когда выкипающие водород-кислород используются вообще для всего на борту, от питания до ориентации.

Также неплохая термодинамика была у Бурана, внезапно - если не ошибаюсь это первая долгоживущая криогенная вундервафля на орбите. Но там был кислород, это попроще. Не помню уже, на сколько там рассчитывали термодинамику, не то на 10, не то на 20 дней, но порядочно - на всё время миссии, в общем. Не то что Шаттл, который даже несмотря на отсутствие криогеники не мог летать в определенное время года - жарко ему было без земной тени, да еще и в некоторых случаях плохело без закрутки для равномерной обжарки со всех сторон.

Но для миссий к Марсу это все тоже хуйня, месяцы криогеника в любом случае не протянет ни при каких ухищрениях выше. Там уже нужны многослойные зонтики, защищающие от солнца, и активное охлаждение - криокулер, отводящий тепло в радиатор. См. например криогенные телескопы, такие как JWST или Спектр-М или ёбы сумрачного японского гения с прослойкой из неонового льда в баке, в таких обычно температуры жидкого гелия и ебанутые двух-трехступенчатые криокулеры.

С водородом и тем более с металоксом конечно нужны не такие сумасшедшие технологии чем с жидким гелием, но без зонтиков всё равно никуда. Тот же боинг рисует орбитальные заправочные хранилища с зонтиками и нулевым выкипанием.
Аноним 19/04/20 Вск 01:50:16 559144477
>>559063
>не позорься, а
Охлол, чмоня, уж кто позорится, так это ti. Ты думал большой кусок английского текста это такой крутой аргумент, что прям всё-всё? Огорчу тебя, дружище.

Вот что ты писал:
>Второй момент - LOP-G можно нормально снабжать коммерческими провайдерами, тяжелых ракет хватает для небольших грузовиков доставляющих эксперименты и мелкие запчасти. А это резко упрощает поддержание станции в рабочем состоянии, не нужно надеяться на единственный SLS. С такой же станцией на низкой орбите луны это не выйдет.

>коммерческие носители
>небольшие грузовики снабжения
Нахуя теперь кидаешь про СЛС и Орион? По-твоему что, "небольшой грузовик", специально разработанный под LOP-G не сможет в 1800 м/с dV на низкую окололунную и обратно? Но сможет в 1000 м/с dV на галоорбиту и обратно? Ясен хуй сможет и туда и сюда, полезного груза процентов на 20 меньше будет, вот и всё.
Так что это ты не позорься, братан. Снабжать можно любой Лоп-Ж, и на низкой окололунной и на гало.
Аноним 19/04/20 Вск 03:39:47 559155478
>>559087
>Для жидкого водорода из-за его крайне низкой температуры тепловой поток через стенку будет в тыщщи раз сильнее.
С херов? Мощность излучения пропорциональна четвертой степени температуры, так что нет особой разницы, что у тебя в баках кипит, кислород или водород. У водорода даже теплота кипения в два раза выше, так что выкипать он будет в два раза медленнее. Вот сжижить его гораздо труднее, но это уже другая проблема.
Аноним 19/04/20 Вск 04:12:46 559160479
>>559155
>С херов?
В уравнение теплопередачи погляди. Туда входит разность температур (а не абсолютная температура) и поверхность теплообмена. У водорода большие проблемы и с тем и с другим:
- в космос отдает тепло неохотно, ибо между 20К и фоном разница минимальная
- в атмосфере принимает дохуя от внешней среды, ибо разница большая
- площадь у водородного бака выше из-за низкой плотности.

Так что да, водород выкипает просто с космической скоростью, если не прибегать к танцам с бубном.
мимо
Аноним 19/04/20 Вск 04:18:39 559162480
>>559160
Алсо, у кислород-водорода еще есть нюанс - трубу сквозь бак пускать тяжело, ибо между ними разница в 70 градусов. Её, внезапно, тоже придется теплоизолировать. Поэтому в водородных ракетах трубу чаще всего пускают снаружи бака, ебясь с крутящим моментом потока перекачиваемой по ней жижи.
Аноним 19/04/20 Вск 10:10:57 559181481
>>559140
>Также неплохая термодинамика была у Бурана, внезапно - если не ошибаюсь это первая долгоживущая криогенная вундервафля на орбите.
Так там буквально холодильник стоял.

Классный псто, малаца.
Аноним 19/04/20 Вск 10:13:34 559182482
>>559162
Почему бы не иметь водород сверху, оксиген снизу? Кислороду освежающий поток водорода будет только на пользу же.
Разве так сильно развесовка влияет в современных машинах, которые могут буквально карандаш на кончике пальца балансировать?
Аноним 19/04/20 Вск 11:36:04 559188483
>>559182
Волород в трубе закипит.
Аноним 19/04/20 Вск 11:39:07 559189484
>>559188
Ах, ёпт, все равно обмазывать, виноват-с.

А как там подвижечки с водородной шугой?
Аноним 19/04/20 Вск 12:21:16 559195485
>>559189
Нет и не будет. Как и вообще водородных ракет в этой стране.
Аноним 19/04/20 Вск 12:22:33 559196486
>>559182
>Кислороду освежающий поток водорода будет только на пользу же.
AMOS-6 уже почуствовал пользу мороженого кислорода
Аноним 19/04/20 Вск 12:26:17 559197487
>>559196
Так це ж очередной дефект СОПВушки.
Аноним 19/04/20 Вск 12:26:56 559198488
pepe crying.gif 1618Кб, 498x490
498x490
>>559195
А ведь были.
Каждый раз как вспоминаю Бураняшу с Энергиечкой, так печалюсь.
Аноним 19/04/20 Вск 12:37:57 559199489
>>559140
>Шаттл, который даже несмотря на отсутствие криогеники не мог летать в определенное время года - жарко ему было без земной тени
Чиво, бля? Или менять наклонение орбиты уже религия не позволяет?
>>559155
>Мощность излучения пропорциональна четвертой степени температуры
>нет особой разницы, что у тебя в баках кипит, кислород или водород
/0
>>559162
>в водородных ракетах трубу чаще всего пускают снаружи бака
Ага, в целой одной
Аноним 19/04/20 Вск 13:01:18 559200490
image.png 3694Кб, 1200x1793
1200x1793
image.png 9500Кб, 3000x2000
3000x2000
image.png 626Кб, 698x1024
698x1024
image.png 5729Кб, 1600x2400
1600x2400
>>559199
>Чиво, бля? Или менять наклонение орбиты уже религия не позволяет?
Шаттл мог менять наклонение на 2-3 градуса.
На произвольные орбиты стартовать он не может, это не одноразовая ракета.
>/0
Очень информативно.
>Ага, в целой одной
Иди в родной /б/, там плюйся своей неконструктивной желчью, как привык, тут тебе не место.
Аноним 19/04/20 Вск 13:43:19 559203491
>>559199
>Чиво, бля?
https://spaceflight.nasa.gov/feedback/expert/answer/mcc/sts-92/10_15_03_59_48.html
>At very high Beta angles (above 65 degrees) the Shuttle can no longer meet thermal parameters so it cannot fly a mission during this time.
>Или менять наклонение орбиты уже религия не позволяет?
Профиль миссии не позволяет. Например станции, к которым он летал (линк выше про МКС говорит). В целом у шаттла было дохуища термодинамических ограничений.

>>559199
>Ага, в целой одной
Common Booster Core дельтовский, шаттл, Энергия, H-II, Saturn, Ariane, Atlas/Centaur, SLS, Vulcan и т.д.
Аноним 19/04/20 Вск 13:45:02 559204492
image.png 460Кб, 319x1050
319x1050
Аноним 19/04/20 Вск 13:48:26 559205493
1929093800.jpg 157Кб, 800x729
800x729
>>559204
А я чет полагал что эти хуйнюшки продольные по бокам это жижепровод, а это демпферы чтоб не плескалось (как они вообще работали?)
Аноним 19/04/20 Вск 13:51:55 559207494
>>559070
>Че обзываешься, сука? Если бы все всё знали, тред бы не имел смысла.

Я на тон обзываюсь. "Ряяя чотко дерзко ядерный реактор иначе несчитово".
Аноним 19/04/20 Вск 13:53:25 559209495
Аноним 19/04/20 Вск 14:46:34 559222496
>>559204
Почему не четко по центру, кстати?
Аноним 19/04/20 Вск 14:53:40 559223497
>>559222
Думаю, чтоб воронка не образовывалась. Ну чтобы газообразного вакуума не хапнуть в трубопровод
Аноним 19/04/20 Вск 15:00:45 559228498
>>559066
И чё, какой же массы солнечные панели нужны, чтобы питать ионники, способные дотащить до Марса дылду тонн в 200 за 6 месяцев?

А ещё, я чет не очень уверен, что ионниками можно успеть тормознуть на орбите Марса.

А ещё, вся эта марсианская одиссея — вилами по воде, и ещё не факт, что там вообще будет отдельный межпланетный орбитер, а не какая-нибудь хуйня типа бфр, которая будет с межпланетной скорости вхуяриваться сразу в атмосферу, не выходя на орбиту.
Аноним 19/04/20 Вск 15:02:09 559229499
>>559228
Можно мощно пердануть в нижней точке пролетной траектории чтоб выйти на высокоэллиптическую, а потом попукивать ионниками в периарии.
Ионники не запрещают использование гептильчика же.
Аноним 19/04/20 Вск 15:19:37 559233500
>>559228
Говоря отвлеченно от лопж и конкретных миссий - баллистический захват и прочие низкоэнергетические траектории позволяют и чисто ионниками долететь, вон Dawn совершенно спокойно крутился вокруг Весты и Цереры, хотя пукалки даже отдаленно не хватило бы на быстрое торможение. А в JIMO хотели на ионниках вообще всю систему Джула облететь. Да, всё это медленно, но гоман и короткие траектории нужны обезьянкам, чтоб не киснуть в бочке посреди нихуя. Груз же может хоть пару лет лететь.
Аноним 19/04/20 Вск 15:26:17 559235501
>>559233
А полет на ИОН ОЧКАХ разве не как раз для перевоза мусора, огурцы разве не на вонючке должны летать?
Аноним 19/04/20 Вск 15:37:40 559239502
Если Мошейник доставит огурцов на МКС, Роскосмос всё?
Аноним 19/04/20 Вск 15:41:03 559240503
>>559239
Pockocmoc не зависит от мошейника ровно никак. Он может "всё" совершенно не завися от мошейника.
Аноним 19/04/20 Вск 15:46:17 559241504
>>559240
Такси же до МКС заказывать перестанут, а это одна из последних скреп.
Аноним 19/04/20 Вск 15:48:08 559242505
>>559239
Тащемта МКС сама всё в ближайшие годы.
Аноним 19/04/20 Вск 15:51:48 559243506
>>559241
Зачем переставать заказывать? У тебя с логикой в порядке?
Аноним 19/04/20 Вск 16:03:30 559245507
>>559203
>Профиль миссии не позволяет
Ну тогда это проблемы конкретной миссии. Что некоторые орбиты недоступны в определённые времена года - правда, что в эти времена вообще летать нельзя - нет.
А вот то, что в XXI веке основная цель полётов оказалась на тех самых запрещённых орбитах - знатный факап.
It is inclined 23.5 degrees from the ecliptic plane
Ну нихуя ж себе. Думал, раза в два меньше.

>Common Booster Core дельтовский
Так.
>шаттл
Нещитова
> Энергия
Нет
>H-II
ОК
>Saturn
Ну предположим
>Ariane
ОК
Центавр - ОК
>Vulcan
Не летал, ещё не известно, как будет.
>SLS
Ну, в принципе, да, но пока не летал
Аноним 19/04/20 Вск 16:04:38 559246508
>>559205
Тоненькие хуйнюшки - трубопровод наддувочного газа
Аноним 19/04/20 Вск 16:08:14 559247509
Баад пидорас!
Аноним 19/04/20 Вск 16:16:42 559249510
154799718116621[...].jpg 45Кб, 495x268
495x268
Аноним 19/04/20 Вск 16:18:04 559250511
>>559249
ладно, ок, я спокоен
спасибо за поддержку, коллеги!
Аноним 19/04/20 Вск 18:10:15 559268512
>>559239
Он уже лет 10 как все.
Сейчас просто будет видимая даже для быдла констатация факта.
Аноним 19/04/20 Вск 18:50:25 559275513
А когда летает водородно-кислородная ракета, водород полностью сгорает или там какой-то гидроксид вылетает?
Аноним 19/04/20 Вск 18:55:03 559276514
>>559275
Да, и гидроксиды, и азотные соединения, и еще дохуя всего, в малых долях.

Сука, я не могу найти это, аноны, на помощь, как загуглить такие данные? Не далече как год назад читал эти листовки с выхлопами разных пар.
Аноним 19/04/20 Вск 19:11:29 559279515
Аноним 19/04/20 Вск 19:18:17 559282516
>>559279
Нет. Синхронизация тоже бывает, но сложно представить как это связано с видео.
Аноним 19/04/20 Вск 19:22:05 559283517
Stsetcutaway.jpg 110Кб, 1200x480
1200x480
1929093800.jpg 157Кб, 800x729
800x729
>>559245
>Ну тогда это проблемы конкретной миссии. Что некоторые орбиты недоступны в определённые времена года - правда, что в эти времена вообще летать нельзя - нет.
На практике более-менее свободно на самом деле нет, там тоже дохуя ограничений, шаттл мог летать только в свободные миссии без целевой орбиты, а к другим аппаратам (основное его применение) мог лишь с дополнительными ограничениями на окно - кроме термодинамики, еще и дохуя сложная баллистика, там и RTLS нужно было обеспечить, и окно приземления, и чего только не.
>А вот то, что в XXI веке основная цель полётов оказалась на тех самых запрещённых орбитах - знатный факап.
Не запрещённые орбиты, а любая целевая орбита в конкретные времена года. Шаттл сам по себе знатный факап. Ну и кстати я уверен что Буран тоже проявил бы себя в этом плане если бы продолжил летать, 100% там бы урезали осетра в практических миссиях, и какая-нибудь хуйня вылезла бы. Ибо по природе это точно такой же переусложненный гибридный крафт уровня всё-в-одном.

>>шаттл
>Нещитова
Че это? На водородном баке же сбоку ТРУБЕНЬ, пик1. По ней херачит кислород, огибая водородный бак, потому как насквозь сложно/тяжело тащить. Вулкан будет юзать Кентавр и там тоже ТРУБЕНЬ. Вот в ACES, который возможно придёт ему на замену, хз как сделано.

>>559246
Не, речь про эти белые трубы, потолще. Из черных кружков начинаются и идут вниз.
Аноним 19/04/20 Вск 19:25:07 559284518
>>559279
На видео они механически связаны, тем и синхронизируются. А орбиты КА не влияют друг на друга.
Аноним 19/04/20 Вск 19:27:57 559285519
>>559283
>там бы урезали осетра в практических миссиях
Не вижу ничего плохого в нарезании рыбы.
>по природе это точно такой же переусложненный гибридный крафт уровня всё-в-одном
НЕТ. Тут самая и прелесть. Сам Буран может и был таким же как Шатал и имел немного в своем репертуаре, но Энергия могла летать отдельно! И это был шин. На нужную траекторию она вывела рекордную массу.

>>Нещитова
>Че это? На водородном баке же сбоку ТРУБЕНЬ, пик1.
Всё щитова, не слушай сельдерей. Это вполне подтверждает тезис про наружный канал для окислителя в баках с водородом.

>Не, речь про эти белые трубы, потолще. Из черных кружков начинаются и идут вниз.
Буран.ру не прощупал? Там навигация черезжопная, но может и найтись инфа при должной сноровке.
Аноним 19/04/20 Вск 19:35:39 559289520
>>559285
Я, естественно, про сам орбитер. Требования неизбежно будут конфликтовать в таком многофункциональном крафте. Такие АКС для всего сразу - это аэрокосмический аналог мемной летающей подлодки. Хотя бы полёт без огурцов предусмотрен, и то ладно.
>Буран.ру не прощупал? Там навигация черезжопная, но может и найтись инфа при должной сноровке.
Как я понял из тамошней диаграммы, это демпфер, чтоб водород не бултыхался. Похоже на трубу из нижней в верхнюю часть бака, наверно там уровень меняется в противовес волне, или что-то такое.
Аноним 19/04/20 Вск 19:37:15 559293521
>>559289
А про урезание осетра-то что там было, лол?
Аноним 19/04/20 Вск 19:40:12 559295522
>>559293
Ну вот слетало - а потом на практической миссии оказалось бы что надо добавить вон ту хрень и эту хрень, оно бы обросло ограничениями, условиями и т.п., в результате параметры бы стопудово отличались от изначальной агитки.
Аноним 19/04/20 Вск 19:41:15 559296523
>>559295
А осётр тут при чём? Впервые слышу такую терминологию.
Аноним 19/04/20 Вск 19:51:42 559298524
>>559276
Ну какого же ты хуя анона дезинформируешь? Оксиды азота какие-то. В выхлопе - исключительно водород и вода!
Все ракетные двигатели работают с избытком горючего. Поэтому после сгорания окислителя (кислорода) часть горючего остаётся. Для двигателя H2/O2 с большой степенью расширения, в выхлопе около 75 молекул воды и 25% молекул H2. Ничего больше нет, абсолютно ничего!
Если двигатель атмосферный, не вакуумный, с коротким, обрезанным соплом и низкой степенью расширения, например 20, то выхлоп горячий, до 1,5-2 тыс. градусов. Это не такая высокая температура, чтобы молекулы H2 и О2 не могли образоваться. Они образуются, но всё-таки есть некая микродоля H и ОН. Нв уровне сотых долей процента. Скорее всего, они объединяются в H20 в факеле двигателя.
Аноним 19/04/20 Вск 20:02:37 559301525
>>559298
Правильно ли я понимаю, что кинетическая энергия преобразуется из сгорания только если горение произошло в камере сгорания, а не в сопле?
Аноним 19/04/20 Вск 20:27:10 559307526
>>559298
>Ну какого же ты хуя анона дезинформируешь? Оксиды азота какие-то. В выхлопе - исключительно водород и вода!
Ебаный ты камень.
1 - азот может присутствовать при вытеснительной системе
2 - азот присутствует в нашей атмосфере и может создавать соединения со свежим выхлопом из-за высокой его энергетики
Говорю же, блядь, целая публикация об этом была. Я тоже по наитию думал, что вода и вода, заебись. а было больше.

>Все ракетные двигатели работают с избытком горючего.
Ложь.
НК-33, он же AJ-26 - избыток окислителя, РД-170 и последователи тоже на избытке окислителя. Китайцы вроде тоже что-то такое пилили.
Ты тот, который утверждал, что у одной ракеты лишь провод окислителя снаружи?
Я тебе уже говорил - иди в /б/, твои беспруфные петушиные выкрики здесь не нужны.
Аноним 19/04/20 Вск 20:32:17 559308527
15866471494101.jpg 407Кб, 1920x1920
1920x1920
Два вопроса.
1) Если человек будет колонизировать Ганимед, то во-первых обязательно под защитой его магнитного поля - а это узкая экваториальная область, а во-вторых скорее всего на стороне, противоположной самому Юпитеру. Но ведь даже это не сведёь ганимединскую бомбардировку излучением всех мастей до по-земному комфортного уровня. Поправьте меня, если я к этому моменту ошибся в рассуждениях. И теперь интересует - а есть ли способы максимально сильно нивелировать низкую гравитацию, высокую радиацию и очень сильную холодрыгу поселения(ий) на обратной Юпитеру стороне Ганимеда где-то в области его пояса?
2) Есть ли игры про космос и планеты наподобии СимСити4? Ну, естественно, инопланетный, и чтобы можно было и выбирать, и настраивать и т.д. Спор и Стелларис - совсем не то. РимВорлд не подходит, ибо очень ограниченно и вообще недоделана и фактически ограничена областью посадкикрушения
Аноним 19/04/20 Вск 21:16:54 559314528
>>559279
Если соотношение масс таково, что в процессе орбитирования центральный объект заметно смещается, эти смещения вполне себе влияют на орбиты всех спутников. Систему Плутона посмотри например, там всё синхронизировано.
Аноним 19/04/20 Вск 21:38:21 559316529
>>559275
>полностью сгорает или там какой-то гидроксид
Горение это и есть реакция соединения вещества с кислородом. Ясен хуй, что исчезнуть вникуда водород не может, что-то должно остаться. А поскольку реакция химическая, а не ядерная, должны остаться те же атомы, что и были, просто в другой комбинации.
Аноним 19/04/20 Вск 21:49:05 559318530
>>559283
>Че это?
Потому что это не ракета а подвесной бак. И ТРУБЕНЬ там не из-за проблем с теплоизоляцией, а из компоновочных соображений.
>Ну и кстати я уверен что Буран тоже проявил бы себя в этом плане если бы продолжил летать
Ну хуй знает. Всё-таки история разработки многоразовых космпланов у нас поболе будет. И на буране хотя бы есть радиаторы.
>Из черных кружков начинаются и идут вниз
Так это не трубы. Это рельсы какие-то. (Видимо, для более равномерного распределения нагрузки)
Аноним 19/04/20 Вск 22:41:14 559328531
>>559318
>Потому что это не ракета а подвесной бак.
Бля, как будто что-то меняется от этого.
>И ТРУБЕНЬ там не из-за проблем с теплоизоляцией, а из компоновочных соображений.
Трубень там от проблем с разностью температур в первую очередь - вынос наружу позволяет сделать её сильфонной без лишней ебли. (не считая оледенения, но это стандартная проблема в криогенных сильфонах) На ранних версиях оранжевого бака была еще одна трубень рядом с ней, противофонтанная, после первых нескольких полётов оказалось что она нахуй не нужна, и её выбросили.

Радиаторы у шаттла тоже есть (ATCS), причем точно там же под створками, просто бурану своих хватало, а ему нет.
Аноним 19/04/20 Вск 22:41:30 559330532
>>559308
1) Радиация не от Юпитера идёт, а от заряженных частиц, разгоняемых магнитным полем, они в направлении от полюса до полюса ебашат по магнитным линиям, и покрывают всё. Нивелировать гравитацию невозможно, если не жить на карусели.

2) https://mars-sim.sourceforge.io/
https://en.wikipedia.org/wiki/Outpost_(1994_video_game)
https://en.wikipedia.org/wiki/Outpost_2:_Divided_Destiny
Аноним 19/04/20 Вск 22:47:48 559333533
>>559318
>ТРУБЕНЬ там не из-за проблем с теплоизоляцией, а из компоновочных соображений.
Доказывай это утверждение.
Аноним 20/04/20 Пнд 02:45:37 559354534
>>559307
ЧЕЕ БЛЯТЬ?!!
какой те нахой "азот из вытеснительной системы"? он как в двигатель попадёт, дурилко?
а про озбыток окислителя и комментировать не буду, ты перепутал окислительный газогенератор и соотношение компонентов топлива, /б/-рат
Аноним 20/04/20 Пнд 03:44:50 559359535
HH47HH34andHH2.jpg 294Кб, 1280x1111
1280x1111
Аноним 20/04/20 Пнд 14:52:42 559445536
>>559354
>он как в двигатель попадёт, дурилко?
Путём растворения в вытесняемой жидкости?
Аноним 05/06/20 Птн 10:42:27 580304537
>>558259
Если прям щя солнце станет 3 км в диаметре с той же массой какие подводные?
Аноним 05/06/20 Птн 13:54:59 580413538
>>558406
за счет чего она распадается и почему одна улетает а не обе падают
Настройки X
Ответить в тред X
15000
Макс объем: 40Mб, макс кол-во файлов: 4
Кликни/брось файл/ctrl-v
Стикеры X
Избранное / Топ тредов